Jump to content

Wikipedia:Reference desk/Science: Difference between revisions

From Wikipedia, the free encyclopedia
Content deleted Content added
Line 499: Line 499:
::I've also heard the alternative version that dogs do this to spread their feaces to mark their territory in tyhe same way that they urinate on every tree in sight. [[Special:Contributions/190.244.186.234|190.244.186.234]] ([[User talk:190.244.186.234|talk]]) 23:46, 29 September 2008 (UTC)
::I've also heard the alternative version that dogs do this to spread their feaces to mark their territory in tyhe same way that they urinate on every tree in sight. [[Special:Contributions/190.244.186.234|190.244.186.234]] ([[User talk:190.244.186.234|talk]]) 23:46, 29 September 2008 (UTC)
:::::Well, yes. To phrase it in a more concise manner: It seems likely that wolves have an instinct to bury their poop. Domesticated dogs do not need that evolutionary adaptation and are therefore losing the instinct...but it's not entirely gone yet. It's possible that the original instinct is to spread the stuff rather than bury it. It's hard to know for sure. From what I've observed in my dogs, they are definitely trying to bury it - but they do indeed give up after a couple of half-hearted scrapes. [[User:SteveBaker|SteveBaker]] ([[User talk:SteveBaker|talk]]) 02:38, 30 September 2008 (UTC)
:::::Well, yes. To phrase it in a more concise manner: It seems likely that wolves have an instinct to bury their poop. Domesticated dogs do not need that evolutionary adaptation and are therefore losing the instinct...but it's not entirely gone yet. It's possible that the original instinct is to spread the stuff rather than bury it. It's hard to know for sure. From what I've observed in my dogs, they are definitely trying to bury it - but they do indeed give up after a couple of half-hearted scrapes. [[User:SteveBaker|SteveBaker]] ([[User talk:SteveBaker|talk]]) 02:38, 30 September 2008 (UTC)
::::::Well actually y'all have no idea what goes on in dogs' or wolves' heads - maybe they all deductively reason out the pros and cons of poop-burial, and it's intelligence that was evolved. But that's just me being a trollish animal-rights person. I mean...not every trait or behavior is directly derived from natural selection...I sure don't see how editing wikipedia increases my chances of getting a mate, but I do it anyways :) --[[User:Bmk|Bmk]] ([[User talk:Bmk|talk]]) 06:25, 30 September 2008 (UTC)
::::::Well actually y'all have no idea what goes on in dogs' or wolves' heads - maybe they all deductively reason out the pros and cons of poop-burial, and it's intelligence that was evolved. But that's just me being a trollish animal-rights person. --[[User:Bmk|Bmk]] ([[User talk:Bmk|talk]]) 06:28, 30 September 2008 (UTC)


= September 30 =
= September 30 =

Revision as of 06:28, 30 September 2008

Welcome to the science section
of the Wikipedia reference desk.
Select a section:
Want a faster answer?

Main page: Help searching Wikipedia

   

How can I get my question answered?

  • Select the section of the desk that best fits the general topic of your question (see the navigation column to the right).
  • Post your question to only one section, providing a short header that gives the topic of your question.
  • Type '~~~~' (that is, four tilde characters) at the end – this signs and dates your contribution so we know who wrote what and when.
  • Don't post personal contact information – it will be removed. Any answers will be provided here.
  • Please be as specific as possible, and include all relevant context – the usefulness of answers may depend on the context.
  • Note:
    • We don't answer (and may remove) questions that require medical diagnosis or legal advice.
    • We don't answer requests for opinions, predictions or debate.
    • We don't do your homework for you, though we'll help you past the stuck point.
    • We don't conduct original research or provide a free source of ideas, but we'll help you find information you need.



How do I answer a question?

Main page: Wikipedia:Reference desk/Guidelines

  • The best answers address the question directly, and back up facts with wikilinks and links to sources. Do not edit others' comments and do not give any medical or legal advice.
See also:


September 24

Underwater urination

Its easy to pee at sea level, but at what depth below the ocean surface would it become impossible due to the external pressure?--79.76.251.108 (talk) 00:00, 24 September 2008 (UTC)[reply]

This is equivalent to asking if a balloon can leak air at under high external pressure. The external pressure pushes on the outside of the balloon (or your bladder) and forces the contents out. So, no, under any pressure in which a human could survive, urination would not be a problem. -- kainaw 00:10, 24 September 2008 (UTC)[reply]
Agreed, as your question stands, its a simple answer. However, if you were in some sort of vesicle which was at STP (standard temperature and pressure), and you urinated into a system at a different pressure, then plausibly sea water could enter your bladder because of the fluid force being higher on the outside. If your skin was "completely impermeable" then it would be impossible to urinate below a certain depth (and even above this depth, you wouldn't be able to fully empty your bladder, but only an amount less than at STP). Sentriclecub (talk) 03:21, 24 September 2008 (UTC)[reply]
I have never had any problem peeing at any depth below the ocean's surface. I don't think it would matter how deep you go. The pressure in the human body (which consists mostly of incompressible liquid) is equal to the pressure of the surrounding environment. Forcing urine out requires no more effort at depth than at the surface.
Now, as Sintriclcub says, if you were in a submarine with an internal pressure of 1 atmosphere, and you wanted to drill a hole through the hull to pee out of, you would have to compete with the water pressure on the outside to force your urine out. ~Amatulić (talk) 20:10, 24 September 2008 (UTC)[reply]
True, your urine would be much more compact inside your body. It would be just as easy. Open system, free equilibrium. Sentriclecub (talk) 09:58, 25 September 2008 (UTC)[reply]

Death under stress

In a previous question, Kurt Shaped Box said that "If an animal is stressed at the time of death (and strangulation tends to be a stressful experience), various chemicals are released into the bloodstream which (negatively) alter the flavour of the meat and cause it to go off more quickly." Is the same true of humans? Can this be used to determine whether a person died under stress or "peacefully in their sleep"? Is it ever used in court? Plasticup T/C 00:53, 24 September 2008 (UTC)[reply]

A few years ago, scientists used tissue and blood samples from deceased deer to demonstrate that the practice of hunting with hounds was a stressful experience for the quarry (see [1]). So, yes - I'd imagine that there would be discernible signs, for those who knew where/how to look. --Kurt Shaped Box (talk) 01:47, 24 September 2008 (UTC)[reply]
Sadly, I just learned this a week ago, but here it goes anyway. These "various chemicals" are called signals. On the surfaces of cells, there exist various glycoproteins called receptors. Different cells can respond to the exact same signal, in completely different ways! The most cited example is norepinephrine. It will cause different results in different cells because of the physiology of the different tissues comprising massive quantities of similar cells. The main response is the "signals" will cause the vast mega-accumulation of the requested proteins. To answer your question, yes, it could be potentially used in forensic evidence if there are few ambiguities. For example, if a stress hormone causes the same response as a hunger hormone, then if the prosecution argues "look at these protein buildups--the person died in anguish!" the defense would counter "look at these protein buildups--the person died hungry". But yes, as long as there's not much ambiguity, then it could be used as forensic evidence. Remember, the "judge" and "jury" interpret expert testimony. I watched 3 consecutive days of the trial of the accountant who shot up his workplace (famous story, dont know his name) but I think you asked a very good question. It shows that you are creative, and I wish all questions were more like yours that relied on creativity and critical thinking. Its no fun to answer questions that just blue-link to the article, or answer homework questions. Sentriclecub (talk) 03:15, 24 September 2008 (UTC)[reply]
An answer and a compliment. What more could I want? Plasticup T/C 04:32, 24 September 2008 (UTC)[reply]
I'm not so sure. For animals, stress is generally a short-lived matter. The deer gets chased - if it escapes, it's back peacefully nibbling the grass within minutes. We humans can get stressed about (for example) the state of the financial market and remain stressed for days or weeks at a time. Should you happen to "die peacefully in your sleep" during an extreme financial crisis, I think your chemistry might look pretty similar to someone who has been stressed by (say) a burglar. We also deliberately stress ourselves - there are human "adrenaline junkies" who jump out of airplanes, ski down cliffs or drive their cars very fast "just for fun". Those people could be the happiest people in the world and yet show all the signs of high adrenaline in their blood stream. So the chemistry might tell you something about the person at time of death - but because humans are so complicated (behaviorally) - it might be almost impossible to interpret the results in any meaningful or useful way. SteveBaker (talk) 00:25, 25 September 2008 (UTC)[reply]
Again, I'm not talking about emotional stress or some other psychological criteria for definition. I'm talking strictly physiological stress, the kind definitively linked to certain molecules which trigger the vast mega-production of proteins inside the cells of various tissue types. Sentriclecub (talk) 10:00, 25 September 2008 (UTC)[reply]

Human body

We humans are built to withstand atmospheric pressure and we do not expand or contract. But if we were to go high in the atmoshere would our body expand? Also, if we were to go tho the bottom of the ocean, would our body contract?--GreenSpigot (talk) 01:03, 24 September 2008 (UTC)[reply]

Your assumption that humans do not expand or contract is not entirely true. The human body is designed to expand and contract as needed. Just ask anyone who can feel a storm coming as joints (especially knees) react to the change in air pressure. -- kainaw 01:19, 24 September 2008 (UTC)[reply]
I think he means that humans don't blow up like balloons. The small change in pressure inside joints is negligible. The only part of the body that really adjusts to pressure is the inner ear. Youth in Asia (talk) 03:00, 24 September 2008 (UTC)[reply]
Humans do expand and contract. Drive up a mountain with an apparatus attached to your body that disallows your ears to pop, you'll see. But the question you ask is really complicated. Humans are an open system, which means every femtosecond your body adjusts to restore equilibrium. Also, re-think your question in terms of everyday scientific terms. This is a question about volume. At the bottom of the sea, your density would increase. (the water would squish you into a smaller version). You would have the same mass (for simplicity, lets skip the microassumptions), and since your volume decreases, and your mass increases, then viola! your density increases. This is an easy question, but if you try to answer it without science, you may as well discuss evolution vs intelligent design. Hope you find this analogy helpful. Sentriclecub (talk) 03:06, 24 September 2008 (UTC)[reply]
(conflict) Excuse me, Youth in Asia, but the human body absolutely blows up like a balloon. It's called breathing, and it affects the chest, abdomen, really the whole torso. Your abdomen also expands to respond to a large meal. The human body is very flexible in terms of changes in volume. As far as the original question goes, the OP may find our articles on Space exposure, Human_adaptation_to_space#Unprotected_effects, and Uncontrolled_decompression#Fallacies enlightening. In short, if exposed to the vacuum of space you will not explode but you may bloat a bit depending on how long you're exposed. If exposure is < 30s you will probably recover completely. Otherwise you will first lose consciousness, then your blood and bodily fluids will begin to boil, resulting in bloating but not rupture of the body because of the strength and flexibility of the skin (see ebullism). CO2 will be released form the blood, resulting in damaging changes in blood pH, and nitrogen will also evolve from the blood forming embolisms. Ultimately, death will result form hypoxia. --Shaggorama (talk) 06:03, 24 September 2008 (UTC)[reply]
I'm pretty certain, that if you are talking about extraterrestrial space, then the human body expands well beyond the limits of survivabilitity. Yeah, step out of the spaceship, without your suit, and you will instantly "expand" to about 1,000,000 times your original size. I think the relevant article is about the partial pressures of the organic compounds comprising your skin cells, and then everything else will follow. But I'll let you have the last word, I max 10 edits to this page, then I take a 1-month vacation. Sentriclecub (talk) 06:48, 24 September 2008 (UTC)[reply]
Our articles disagree with your expanding hypothesis. --mboverload@ 07:00, 24 September 2008 (UTC)[reply]
The difference caused by stepping out of a spacecraft is the same as it caused by rapidly ascending through 10 metres of water. If you try and hold your breath, your lungs might "explode" but most of your body will remain intact - skin is capable of surviving a change of 1 atm. --Tango (talk) 12:47, 24 September 2008 (UTC)[reply]
Okay, let's summarise and separate out the hard science from the misinformation and speculation in the above responses:
  • High pressure: The human body is, chemically, a dilute solution of proteins and lipids in water. The bulk modulus of water is about 2x109 Pa. So to reduce the volume of a given mass of water by, say, 1% requires a pressure of 2x107 Pa or about 200 atmospheres. In other words, the body's cells and organs are, for practical purposes, incompressible. Saturation divers live and work in pressurised habitats at pressures of more than 5 atmospheres for days at a time. Ultra-deep divers at depths of more than 300 metres experience pressures of 30 atmospheres or more. The most dangerous effects of high pressure on the human body are associated with breathing gases under pressure, not with compression of body parts - see nitrogen narcosis and high pressure nervous syndrome.
  • Low pressure: Our article on uncontrolled decompression says "accidents in space exploration research and high-altitude aviation have shown that while vacuum exposure causes swelling, human skin is tough enough to withstand a drop of one atm. This assumes that the person doesn't attempt to hold their breath (which is likely to cause acute lung trauma), the limiting factor on consciousness then being hypoxia after a few seconds". So it is lack of oxygen that kills in vacuum, not the low pressure itself. Soyuz 11 depressurised slowly in orbit, shortly before re-entry, and the crew suffocated, but they did not explode. When Joseph Kittinger's glove seal failed during his Excelsior III balloon ascent, his hand was exposed to a pressure of 1/100 of an atmosphere for several minutes - this caused pain but no long-term damage. Our article on space exposure says that the maximum survivable time for exposure to vacuum is around 90 seconds.
  • Rapid decompression: a quick transition from high pressure to a lower pressure is the most dangerous scenario - see uncontrolled decompression. This not due to the expansion of body parts themselves but because of the expansion of gas in various places within the human body. Going from 1 atmosphere to very low pressure in less than half a second does not allow time for the lungs to empty of air, so causes lung damage. The effects of explosive decompression from 9 atmospheres to 1 atmosphere are graphically described in our account of the Byford Dolphin diving bell accident. Divers who surface too quickly can experience decompression sickness, due to the release of gases dissolved in body liquids and tissues.
Hope this help. Gandalf61 (talk) 13:08, 24 September 2008 (UTC)[reply]
Addendum to "Low Pressure": Its not just the lack of oxygen in the vacuum that kills you. The low pressure causes your blood to boil and so you lose what little oxygen may already be dissolved in your blood. The low pressure robs you of oxygen you have already inhaled! Also, I think the main article to point to should be space exposure. Otherwise, nice summary --Shaggorama (talk) 02:55, 25 September 2008 (UTC)[reply]

the first fortified peanut butter that is not plumpy nut

I am looking for any information on fortified peanut butter. Not the Plumpy Nut, but anything having to do with fortification, enhancement, any additions to etc. —Preceding unsigned comment added by G1963 (talkcontribs) 02:34, 24 September 2008 (UTC)[reply]

Density

I feel like this question is like the "which weighs more, 1 lb of bowling balls or 1lb of feathers", but I'm not sure.

"If you were to find the mass of 20 mL of seawater, would it be heavier, lighter, of the same as 20 mL of distilled water? And what does this mean in terms of density?"

I thought that the masses of each would be the same, and that the density of the seawater was more, but my friend told me I had it backwards. Is he right, or am I? —Preceding unsigned comment added by 69.16.88.147 (talk) 02:39, 24 September 2008 (UTC)[reply]

Without saying anything about seawater, we can reduce the two questions to one: if the a volume of X has more mass than the same volume of Y, then X is more dense. That's just the definition of density. To decide which substance is denser requires knowledge of the composition of seawater and what that means for density (or else looking it up). The important bit here is that adding a solute (e.g., salt) to a solvent (e.g., water) often increases the density because the molecules of the solvent clump up around those of the solute, reducing their volume. --Tardis (talk) 02:56, 24 September 2008 (UTC)[reply]
Agree with the first answer. Furthermore, think of buoyancy. When you lay in salt water, the buoyancy is different. You can float easier. Secondly, mythbusters did a show on quicksand. The result, is that you float extremely well, you have about 30% of your body above quicksand. Also, the ethereal air around us is very non-dense. When a skydiver jumps from a plain, that is like "sinking" because he is more dense than the air around him. Sentriclecub (talk) 03:00, 24 September 2008 (UTC)[reply]
Don't skydivers jump onto the plains, rather than from them? (sorry). :) -- JackofOz (talk) 05:38, 24 September 2008 (UTC)[reply]
Yeah you caught me, in the language of mathematics, there are no homophones. I'm a regular at the math ref-desk, only came here to post a question, but got too lazy. I'd be happy abolishing letters and using numbers only to communicate in writing. In which case, prime numbers could serve as vowels. I think I goofed because I think of the word plane as a mathematical term, oh well, I goofed, I should enjoy it. Sentriclecub (talk) 06:42, 24 September 2008 (UTC)[reply]
8 1, 8 1. -- 10 1 3 11 15 6 15 26.
9 & 7 5 20 & 9 20 Sentriclecub (talk) 10:03, 25 September 2008 (UTC)[reply]
I'd really like to see 1 lb of bowling balls! SteveBaker (talk) 23:50, 25 September 2008 (UTC)[reply]

Progressivity of electrical loads

If we attach a variable voltage source across a load, we will typically produce a fixed current for each voltage and obtain a current function . With a few exceptions, the function is strictly increasing, and if the load is unpowered clearly . For what broad classes of unpowered loads is the function convex, concave, or neither? (A pure resistance/reactance, of course, is the only convex and concave load.) Are there any reasonable components for which the current function is multivalued? DC and AC (considering the magnitude of current; see power factor) both interest me. --Tardis (talk) 03:22, 24 September 2008 (UTC)[reply]

A voltage source across a forward biased diode would produce negligible current until a certain voltage was applied (.7 volts for a silicon diode) then sharply increasing current for higher voltages, thus perhaps "concave" although the applicability of "concave" and "convex" is questionable. For a classic resistor it would be linear. Edison (talk) 05:48, 24 September 2008 (UTC)[reply]
Im afraid that what you say about diodes is entirely wrong, Edison. A semiconductor diode always has an exponential relationship of current to applied voltage (neglecting the small reverse leakage current). The current is given (approximately) by I= Io exp(qV/kT). This sort of poppycok bandied about by many people about diodes behaving as some sort of voltage dependent switch does nothing to aid understanding of electronics and in fact can lead to many poor designs in circuitry. The article Diode has a grossly misleading diagram of the I V characteristics of a semiconductor diode, implying as it does, that there is some magical transition between ON and OFF. There is NO such transition. (see the equation) It is worth saying also that the equation shows that the voltage across two similar diodes will only be equal if thier currents are equal. Hence the operation of so called bandgap references. Lecture over!--79.76.251.108 (talk) 17:31, 24 September 2008 (UTC)[reply]
Things like fluorescent lamps have negative resistance so are convex to a degree. Other things like normal incandescent lamps or real resistive loads have concave function as the higher temperature lead to higher resistance. --antilivedT | C | G 08:56, 24 September 2008 (UTC)[reply]
Anything with negative resistance is automatically "neither" since it can't have it at and can't be decreasing forever. The diode and the incandescent lamp are the two examples I had been able to come up with; I should have mentioned them. That the diode is only convex when forward biased reminds me also that positive and negative voltage need to be considered separately (anything unpolarized must have an odd current function and can't be nontrivially convex/concave everywhere). Thanks for the input — any other ideas? --Tardis (talk) 13:26, 24 September 2008 (UTC)[reply]
Have you seen our article on Negative resistance. It pretty much covers everything you need to know.--GreenSpigot (talk) 05:01, 25 September 2008 (UTC)[reply]
I have; I was just looking for more examples, to get a feel for the most common cases. --Tardis (talk) 23:36, 28 September 2008 (UTC)[reply]
Hmm. Have you seen Diac?--GreenSpigot (talk) 22:29, 1 October 2008 (UTC)[reply]
Also, what about thermistors?--GreenSpigot (talk) 22:31, 1 October 2008 (UTC)[reply]

Lag time and planetary alignment issues regarding laser transimission between Mars and the Moon/Earth

I'm doing research for a web comic, and I'd like to know, roughly:

1- The lag time for laser communications between the Moon/Earth and Mars.

2- The Sun is between Mars and the Moon/Earth for X amount of time. Time of year would be great but not necessary.

Also, I'm assuming that all lasers travel at the speed of light or thereabouts i.e. specific wavelength doesn't matter. Please don't trouble yourself looking for a formula(s) - I'm afraid physics equations are a bit beyond my reach.

Again, I'm looking for general estimates- please don't go through a lot of trouble calculating. Thank you so much!

Sevenzark 7 (talk) 05:36, 24 September 2008 (UTC)sevenzark_7[reply]

speed odf light 0.3 million km/s so divide the distances in the Mars and Moon article (normally given in million km) by 3 and you have the seconds. Easy to do it! But for the moon the estimation would be one second for one direction and there and back 2 seconds.--Stone (talk) 06:51, 24 September 2008 (UTC)[reply]
For Earth/Mars communication it varies depending on the time of year on both Earth and Mars. If we're both on the same side of the sun, I think it can get as low as three and a half minutes, or if they're on opposite sides of the sun it can get as high as twenty minutes. Roughly. APL (talk) 12:46, 24 September 2008 (UTC)[reply]
And starting around the mid-November, NASA will loose communication with the rovers/lander for 2 weeks, while Mars is directly behind the Sun. [2][3] -- MacAddct1984 (talk &#149; contribs) 13:34, 24 September 2008 (UTC)[reply]
I would imagine that laser communication from Earth to Mars would be really tough when they are more or less opposite each other with the Sun in the middle. Imagine pointing your super-sensitive light detector anywhere close to the sun and looking for a bajillionth of a candela of laser light energy. It's worth saying that the speed of light (which is the speed of the laser) and the speed of radio waves is exactly the same - so there is no particular advantage (or disadvantage) in using a laser versus a more conventional radio link. SteveBaker (talk) 00:11, 25 September 2008 (UTC)[reply]
(See bit rate :) Saintrain (talk) 01:09, 25 September 2008 (UTC)[reply]
The time of year during which the Sun will be directly between Earth and Mars is different every year, so it depends which year your comic story is supposedly happening. You could predict it, though, based on the future date and reasonnably simple calculations and MS excel. Some years the Sun will never be directly between the 2. --Lgriot (talk) 07:09, 25 September 2008 (UTC)[reply]

AWESOME! Thank you folks! Thanks SteveBaker I was wondering about radio waves. MacAddct1984 and Lgriot thanks for the date info! Sevenzark 7 (talk) 21:29, 28 September 2008 (UTC)sevenzark_7[reply]

Cell phone radiation and hard drives

Does cell phone electromagnetic radiation cause any harm (file corruption, data loss, etc.) to computer magnetic hard drives? If one were to put five cell phones (in use) on top of an external hard drive, would it do anything? —Lowellian (reply) 05:40, 24 September 2008 (UTC)[reply]

Short answer--no. Cell phones use low energy electromagnetic waves. If you have ever been in the airport and your computer was passed through a x-ray, that is about 10,000 times more energy per photon. Ionizing radiation is hopefully a blue-link. 99.9999999% of peoples computers survive the x-ray machine. However, the long answer is that my TV got slightly miscolored when I put my cell phone up to it. It needed a degauss button, but its an old tv, like 10 years old. But honestly, I'm supposed to be scientific, but I partly believe that cell phones shouldn't be held up to the ear when talking through them. Afterall, isn't the question about brain cancer the big elephant? I'm 24, and I own no cell phone! Hurray for me. Also, the neat part of your question... put five cell phones, is that electromagnetic radiation is not additive. In other words, if an apparatus is 60% of the energy required to emanate ionizing radiation, then a thousand of these operating concurrently will still not be able to emanate ionizing radiation. In the chemistry of photons... 50 joules + 50 joules + 50 joules + 50 joules does not equal 200 joules, but instead simply equals 50 joules. See photoelectric effect. Don't take my answer literally, I'm just regurgitating stuff from textbooks, i still find it hard to believe, and yet fascinating! Sentriclecub (talk) 06:34, 24 September 2008 (UTC)[reply]
I'm still somewhat confused though. I've tried holding up a cell phone in use to a stereo system and a television set, and it produces MAJOR audio and visual distortions, to the point that the audio/video is unlistenable/unwatchable. Considering that it can do so much to a stereo system and a television set, why would it by contrast do so little to a hard drive?
Lowellian (reply) 07:32, 24 September 2008 (UTC)[reply]
That's because the wires in your sound system acted as an antenna, receiving the signal of your cell phone, which is then amplified by your amps and made audible through your speakers. Not sure about video though since I have never experienced that before. Without the amplifier part (which amplified the few milliwatts of power from your cell phone to tens/hundreds of watts to drive your speakers) it simply does not have enough energy to disrupt your hard drive. --antilivedT | C | G 08:46, 24 September 2008 (UTC)[reply]
But if one's computer speakers are right next to one's computer (since they are, after all, computer speakers), by what you're saying, doesn't this then mean that these speakers will amplify the cell phone radiation right next to the hard drive? —Lowellian (reply) 04:12, 25 September 2008 (UTC)[reply]
Computer speakers have amps built into them (notice how they need mains to operate?) as line level signals from your sound card is simply not enough t drive the speakers. Try putting a cell phone next to a proper speaker with no amplification: you won't hear a single thing. --antilivedT | C | G 06:08, 25 September 2008 (UTC)[reply]
In addition, hard drives are completely encased in thick metal, which has Faraday effect to block a lot of EM radiation you can throw at em. Hard drives are designed to be extremely hard to damage - and very much succeed. They are made from solid aluminum (or something else) and are for all practical purposes dust-proof. They can protect the data they hold even when thrown to the ground or dropped from large heights (I think modern HDD can withstand 300g's of shock). With proper data recovery techniques data can be recovered from hard drives that have been underwater for days or in an intense fire. Running hard drives while open or with fingerprints even work (I've done it - although I wouldn't recommend it) Also, even pretty powerful magnets won't do jack to them. To properly degauss a hard drive you'll need a professional degausser (10 thousand dollars). Some lowly EM radiation isn't going to hurt it. --mboverload@ 06:54, 24 September 2008 (UTC)[reply]
I would question whether the thick metal of the average hard drive is effective as a Faraday cage, since it has gaps in it that enable an electromagnetic pulse to easily fry a hard drive.
Lowellian (reply) 07:28, 24 September 2008 (UTC)[reply]
Faraday cage with holes will still block out electromagnetic waves up to a certain frequency. If I'm not mistaken the highest cell phone frequency is 2100Mhz, which has a wave length of roughly 14 centimetres (5 and a half inches). I'm not sure about how small the gap has to be in relation to the wave length in order to block out that wave but unless you have holes 14 cm in size (bigger than your whole hard drive) it's probably blocked by the casing. It's the same principle the screens work in your microwave. --antilivedT | C | G 08:46, 24 September 2008 (UTC)[reply]
No. [4] - QED! SteveBaker (talk) 00:05, 25 September 2008 (UTC)[reply]
No, not quite QED. I actually thought about this particular argument before I asked the question; the reason that cell phones having internal hard drives does not prove that cell phones radiation doesn't affect hard drives is because presumably, even in the case that cell phone radiation did affect hard drives, a manufacturer might still put an internal hard drive inside a cell phone after taking care to specially shield this internal hard drive in a way that normal computer hard drives are not shielded. (I'm not saying that cell phones would harm hard drives; I'm just saying that this particular argument doesn't prove that cell phones wouldn't.) —Lowellian (reply) 04:10, 25 September 2008 (UTC)[reply]
Perhaps you should take a phone apart then :p. Compared to the millimetre thick hard drive enclosures the metal sheets inside cell phones for shielding are diminutive (not that thickness matter anyway). --antilivedT | C | G 06:08, 25 September 2008 (UTC)[reply]

four acceleration

the wikipedia article on 'four acceleration' states that the acceleration felt by an accelerating object is the derivative of proper velocity with respect to proper time. most everything else that I have read including other articles on wikipedia state that it is the derivative with respect to coordinate time. Em3ryguy (talk) 05:57, 24 September 2008 (UTC)[reply]

Proper time sounds right to me - then it's the acceleration that the object would actually feel. Can you give an example of an article that defines it wrt coordinate time? --Tango (talk) 12:37, 24 September 2008 (UTC)[reply]
thats what I thought too at first. but I did some figuring and the acceleration would indeed seem to be alpha=a*gamma^3. a is regular coordinate acceleration. the derivative of proper velocity (v*gamma) with respect to coordinate time equals alpha (according to the online calculator I used). the article on proper acceleration clearly and unambiguously defines it that way. http://en.wikipedia.org/wiki/Proper_acceleration#Viewed_from_a_flat_spacetime_slice Em3ryguy (talk) 20:07, 24 September 2008 (UTC)[reply]
I'm confused, where is the article that disagrees? --Tango (talk) 20:10, 24 September 2008 (UTC)[reply]
http://en.wikipedia.org/wiki/Four-accelerationEm3ryguy (talk) 20:21, 24 September 2008 (UTC)[reply]
Sorry, what article disagrees with that article? Apart from the strange fact that we have an article on 4-acc and on proper acc when they seem to be defined to be the same thing, I don't see any contradiction... --Tango (talk) 21:53, 24 September 2008 (UTC)[reply]
Actually, they're not quite defined to be the same - proper acceleration seems to be defined as a 3-vector, and 4-acc is that 3-vector with a null time component. Is that standard? I would have just called the 4-vector "proper acceleration" and avoided 3-vectors completely... --Tango (talk) 21:56, 24 September 2008 (UTC)[reply]
the article on four acceleration clearly states ' four-acceleration is a four-vector and is defined as the change in four-velocity over the particle's proper time'. the article on proper acceleration clearly defines proper acceleration as dw/dt. that is the derivative of proper velocity with respect to coordinate time.(four velocity and proper velocity being more or less the same thing) http://en.wikipedia.org/wiki/Proper_acceleration#Viewed_from_a_flat_spacetime_slice Em3ryguy (talk) 22:16, 24 September 2008 (UTC)[reply]
which contradicts the statement at the top of that article that says " The proper acceleration 3-vector, combined with a null time-component, yields the object's four-acceleration" which must be wrong. Em3ryguy (talk) 01:30, 25 September 2008 (UTC)[reply]
the article on four acceleration attempts to show that you can use it to calculate the instantaneous (and therefore infinitesimal) change in velocity. but since coordinate velocity, proper velocity, and rapidity are all the same at low speeds then I would think that any of them could do the same. but my math isnt good enough for me to be sure. thats why I'm asking you. Em3ryguy (talk) 20:19, 24 September 2008 (UTC)[reply]
if a stationary observer measures A's coordinate velocity to be v and B's velocity to be v+dv and both start at the origin at t=0 then at t=1 B's position is (v+dv,1). simply transform coordinates to A's frame to get (g(v+dv-v),g(1-v(v+dv))). the infinitesimal velocity from A's point of view is therefore g(dv)/g(1-v^2-vdv). which equals dv*gamma^2. divide the infinitesimal velocity by infinitesimal proper time [d(proper time)] to get acceleration from A's point of view. alpha=a*gamma^3.Em3ryguy (talk) 20:44, 24 September 2008 (UTC)[reply]
you might also want to know that dgamma/dt=v*a*gamma^3 (again according to the online calculator 1/sqrt[1-((v[t])^2)] http://calc101.com/webMathematica/derivatives.jsp#topdoit).Em3ryguy (talk) 20:58, 24 September 2008 (UTC)[reply]
you can also check that the derivative of proper velocity with respect to coordinate time is a*gamma^3. v[t]/sqrt[1-((v[t])^2)] http://calc101.com/webMathematica/derivatives.jsp#topdoit Em3ryguy (talk) 21:19, 24 September 2008 (UTC)[reply]
and of course. d(coordinate time)/d(proper time)=gamma. Em3ryguy (talk) 21:19, 24 September 2008 (UTC)[reply]
I don't follow what you're trying to show there, however it appears you are making a mistake with your coordinate transform. Relativistic velocities don't add simply like that, you need the velocity-addition formula. --Tango (talk) 21:53, 24 September 2008 (UTC)[reply]
I'm not adding velocities. its a coordinate transformation. if the coordinate in one frame is (d,t) then the coordinates (d',t') in a second frame moving at v is (g(d-vt),g(t-vd)). (assuming that their origins coincide at t=0) Em3ryguy (talk) 22:16, 24 September 2008 (UTC)[reply]
What is g? And that's not what you wrote, you had "v+dv-v" which is adding velocities (well, subtracting them actually, but that makes no difference). --Tango (talk) 23:55, 24 September 2008 (UTC)[reply]
g is gamma for coordinate velocity v. (d',t')=(g(d-vt),g(t-vd)) = (g(v+dv)-v(1),g(1-v(v+dv)) assuming c=1 Em3ryguy (talk) 00:06, 25 September 2008 (UTC)[reply]
gamma for velocity v is of course 1/sqrt(1-v^2) assuming c=1 Em3ryguy (talk) 00:52, 25 September 2008 (UTC)[reply]
I think I understand now. the 'a' in the article on four acceleration is the coordinate acceleration as measured by the nonaccelerating observer. only if that nonaccelerating observer is moving at the same instantaneous speed of the accelerating object does it equal the acceleration felt by the object (but, I think, thats true of coordinate velocity, proper velocity, and rapidity since they are all the same at low speeds). in all other cases it does not. the article probably isnt wrong but its certainly confusing. Em3ryguy (talk) 22:38, 24 September 2008 (UTC)[reply]
the article should read 'Therefore, within that co-moving inertial reference frame the four-acceleration is equal to the proper acceleration that a moving particle "feels"'. —Preceding unsigned comment added by Em3ryguy (talkcontribs) 17:25, 25 September 2008 (UTC)[reply]
proper acceleration on the other hand, as I understand it, should always be the same for all observers. just as proper time and proper distance are the same for all observers. Em3ryguy (talk) 22:40, 24 September 2008 (UTC)[reply]
the derivative of rapidity with respect to proper time is the proper acceleritaon. that is also clearly defined in the article on proper acceleration http://en.wikipedia.org/wiki/Proper_acceleration#Viewed_from_a_flat_spacetime_slice Em3ryguy (talk) 00:39, 25 September 2008 (UTC)[reply]
http://en.wikipedia.org/wiki/Lorentz_factor#RapidityEm3ryguy (talk) 23:35, 25 September 2008 (UTC)[reply]

I have edited the pages in question to what I believe is a less confusing form. I would appreciate it if someone would double check my work. Em3ryguy (talk) 01:43, 26 September 2008 (UTC)[reply]

Do candles melt when put in a microwave oven?

This question has been bothering me and my roommates during the whole summer while sitting on our balcony. Enjoying a good wine we were contemplating on the candles that were burning on our table: Do candles melt using microwave radiation? I voted for no, because candles are made out of wax and wax is not influenced by microwave radiation. A microwave oven utilizes the polarity of molecules, such was water, to generate heat inside the object that is inside the oven. As I have found out on Wikipedia, candles are made of paraffin wax, meaning a mixture of alkane hydrocarbons with a length between 20-40 C-Atoms. I am not a chemist, but I would guess that those long alkanes do not have a dipole moment? So, is my explanation correct? Ced22 (talk) 08:43, 24 September 2008 (UTC)[reply]

Having just put a small tea-light into a microwave oven on full strength for 30 seconds I can report that the candle did not melt nor did it feel any warmer than when it went in. Not very scientific but it's a start. Richard Avery (talk) 11:09, 24 September 2008 (UTC)[reply]
On the contrary, that is very scientific. Confronted with a hypothesis (candles heat up in a microwave) you made a prediction (your tea light would heat up in a microwave), tested it (put the candle in the microwave) and used the results to reject the hypothesis. My man, what you did was the definition of science! Plasticup T/C 23:45, 24 September 2008 (UTC)[reply]
Science is not about ad hoc experimentation. --98.217.8.46 (talk) 14:54, 25 September 2008 (UTC)[reply]
Science is all about planned experimentation. Do you mean to imply that Richard Avery's was ad hoc? --Kjoonlee 23:57, 26 September 2008 (UTC)[reply]
I think it's just what Karl Popper would have done. William Avery (talk) 20:28, 25 September 2008 (UTC)[reply]
Thanks very much for doing the experiment, Richard. We do not own a microwave oven, that's why we have not tried this test. Apparently the results support my theory. I did some further research, apparently alkanes exhibit very low polarity.Ced22 (talk) 11:36, 24 September 2008 (UTC)[reply]
Presuming this is an ordinary microwave that you don't wish to break, you might want to be careful with this sort of experiment. If your running your microwave oven with only something which does not absorb microwaves, you're basically running it without anything which as our article mentions, could cause you to burn out the magnetron. While this probably won't happen in 30 seconds, you should try it for too long. The way to avoid this would be to put a glass of water or something similar while doing the experiment Nil Einne (talk) 18:42, 24 September 2008 (UTC)[reply]
A microwave can only heat polar compounds, water being ideal. Candles are made mostly of long chain alkanes (see paraffin) that are certainly nonpolar, so they are unaffected by microwaves. Dielectric constant is essentially a measure of polarity. --Russoc4 (talk) 14:02, 24 September 2008 (UTC)[reply]
That would be Dielectric constant, or relative static permittivity. jeffjon (talk) 15:29, 24 September 2008 (UTC)[reply]

It's probably also worth noting that paraffin wax is, essentially, just a shorter version of the same sort of molecules that make up polyethylene, and polyethylene is widely used as a low-loss dielectric in many electrical applications. "Low loss", in this case, means that it absorbs very little energy from electrical fields.

Atlant (talk) 17:38, 28 September 2008 (UTC)[reply]

Thanks for all the clarifications and comments, guys! Ced22 (talk) 08:55, 29 September 2008 (UTC)[reply]

Floodwater in the bathroom

There was a big flood in Sueca (Com. Valenciana) last night due to rainfall, and some people said on the TV that they had floodwater coming up through the toilet etc. in the bathroom in flats higher up (not groundfloor). I live in a 3rd floor flat: could floodwater theoretically come that far up through the water system in the case of a flood, where exactly does it comes out and how could you stop it? Thanks for info, --AlexSuricata (talk) 11:00, 24 September 2008 (UTC)[reply]

If floodwaters are backing up through the sewers, then the way to shut that off would be to close a sewer valve if one exists or otherwise disconnect your outflow pipes. — Lomn 13:36, 24 September 2008 (UTC)[reply]
If you look out the third floor window and notice floodwater above the level of the third floor toilet, then it might be expected to rise out of the toilet. If the flood water is far below the level of the toilet, it's hard to explain why water would rise far above the level of the source (the floodwater) without work being done, and such a phenomenon would allow a perpetual motion machine consisting of a turbine to generate power from the fall of the water back to the level of the floodwater. If the drain were clogged, and the toilet were flushed, it would overflow, with or without a flood. If the building drain were somehow shutoff to prevent floodwater entering the building, and water were entering the drains from the plumbing pipes due to toilets flushing or water going down drains, or if somehow water collecting on the roof were overflowing into floor drains in the top floor, then water could overflow from third floor toilets. Edison (talk) 15:50, 24 September 2008 (UTC)[reply]
Floodwater can and does emerge from toilets that are higher than the level of the source. It happens when flowing water creates a surge of pressure in pipes. It has been known to happen from sea tides in the early days of Seattle, when toilets drained directly into the sea. It's the same principle as described in blowhole (geology). ~Amatulić (talk) 19:59, 24 September 2008 (UTC)[reply]
I do not believe that tides would cause such a "pressure wave" which would cause water to rise from a thrid floor toilet, presumably many feet above the water level. Consider that building codes (in places I am familiar with) toilets be connected to an upvent as well as a drain, so any pressure wave should go up that upvent to the atmosphere, and the water in the trap should stay in the trap. We have noted here previously, however, that extreme winds have caused pressure variations causing visible level changes in the toilet due to the pressure differential in the bathroom and at the rooftop terminus of the upvent. I just cannot envision the water rising up the drain pipe tens of feet above the level of the floodwater. The system of drains I would expect to dissipate the transient shock of rising water level and prevent the spike of energy required for such a slosh. Edison (talk) 22:22, 24 September 2008 (UTC)[reply]
Wouldn't this need to be considered as a momentum problem? Fluid is flowing downward in the sewer pipes from higher areas. That fluid has a definite momentum. At the same time, fluid is backing up from the overload in lower areas, thus flowing backwards with some lower momentum. When these flows meet, the momentum must be dissipated, presumably via a pressure surge upwards in whichever local pipes are handy.
And of course, if it's a rainwater flood and the rain is falling up on the hills and entering the sewer, if you're in the valley, expect the water to seek its own level, which will be above your toilet. Franamax (talk) 15:40, 25 September 2008 (UTC)[reply]

'foods that promote cancer and how they promote it'

i would like a detailed analysis of foods that promote cancer and how each of these foods help promote this disease41.219.253.218 (talk) 11:59, 24 September 2008 (UTC)[reply]

As in foods that are Carcinogenic#Carcinogens_in_prepared_food? 194.221.133.226 (talk) 12:47, 24 September 2008 (UTC)[reply]
You may want read the whole article. Carcinogens act in a variety of ways as mentioned there. And there are so many different carcinogens and potential carcinogens, many of which may occur in food that any complete detailed analysis is liable to be many, many pages long Nil Einne (talk) 18:24, 24 September 2008 (UTC)[reply]

Location of seminal vesicles

Is the seminal vesicles located between the rib cage and spine ? —Preceding unsigned comment added by 86.96.226.14 (talk) 12:14, 24 September 2008 (UTC)[reply]

Based on the diagrams in our cleverly named seminal vesicles article, I'd say no. --LarryMac | Talk 12:29, 24 September 2008 (UTC)[reply]


Rehanrazak (talk) 15:01, 24 September 2008 (UTC)[reply]

Whats happens when Global Warming comes to apex?

Does another ice age start on earth? If so, how will that happen? What do you think that most countries (ie. USA, UK, etc) would do? --Anilmanohar (talk) 13:05, 24 September 2008 (UTC)[reply]

This is largely beyond the scope of the Reference Desk, as we have no way of knowing what or when the peak of global warming will be. As such, no meaningful prediction of international response can be made. As for ice age trends, it's certainly the case that Earth has historically cycled between warmer and cooler periods. Whether human-affected warming is sufficient to break the cycle is again a question we cannot reliably answer. — Lomn 13:34, 24 September 2008 (UTC)[reply]
Venus might be a good model. Plasticup T/C 15:48, 24 September 2008 (UTC)[reply]
Note that climatically we currently are in an ice age, as there is permanent ice in the Arctic and Antarctic. We are in a slightly warmer interglacial, but the real hothouse mode of Earth is a lot warmer. On the plus side, that means that a complete runaway effect as on Venus is quite unlikely. But on the negative side, that means that we may push Earth into a very much warmer and very different state. --Stephan Schulz (talk) 18:13, 24 September 2008 (UTC)[reply]
Global warming doesn't exist. It's all related to sunspots and solar activity. See Maunder Minimum. 31306D696E6E69636B6D (talk) 13:30, 29 September 2008 (UTC)[reply]
Isn't it kind of weird that what you said is an opinion? This is science, for god's sakes! Mac Davis (talk) 23:44, 29 September 2008 (UTC)[reply]

the charge carried by 100 electrons

write in scientific notation the charge carried by 100 electrons —Preceding unsigned comment added by 76.189.131.73 (talk) 19:24, 24 September 2008 (UTC)[reply]

Done. Not sure it helps, since you can't see my notebook. Why don't you try it now? DMacks (talk) 19:32, 24 September 2008 (UTC)[reply]
1.2 * 10^-7 steves, this probably won't help either since you didn't specify the units and I just made these units up. (1 steve = 1.2 *10^-9 times the charge of an electron) You might want to try electron -- Mad031683 (talk) 19:48, 24 September 2008 (UTC)[reply]
I get about 8.3 × 1010 steves...are you sure your calculations are correct? :) --WikiSlasher (talk) 11:49, 1 October 2008 (UTC)[reply]

a wire carries 2.496E17 electrons

a wire carries 2.496E17 electrons (2.496 x 10^17) determine the charge [c.j] —Preceding unsigned comment added by 76.189.131.73 (talk) 20:04, 24 September 2008 (UTC)[reply]

Done. Algebraist 20:09, 24 September 2008 (UTC)[reply]
You may wish to read electron, specifically for finding the charge of an electron. —Cyclonenim (talk · contribs · email) 20:34, 24 September 2008 (UTC)[reply]

25A fuse

a fuse is rated at 25A.Will it fail if 3600C pass through it in 3 minutes —Preceding unsigned comment added by 76.189.131.73 (talk) 21:17, 24 September 2008 (UTC)[reply]

Check out the definition of current. --Tango (talk) 21:26, 24 September 2008 (UTC)[reply]
Q=It should be helpful--GreenSpigot (talk) 21:40, 24 September 2008 (UTC)[reply]
There is not really enough information to answer the question, if you want to be picky, since it is not specified whether the current flows at a constant rate. Far less charge than 3600 coulombs could blow the fuse if it passed through it in a very short while. If the current source were a large capacitor, most of the 3600 coulombs might flow in the first second. The question is also deficient in not specifying the time/current characteristics of the fuse, since a fuse is not a little magic machine that blows instantly when anything above its nominal rating passes through it. Some 25 amp fuses could carry significantly more than 25 amps for a short while, and others with different time curves would blow quickly at the same current overload. A fuse rated 25 amps should not blow if the current never exceeded 25 amps. If it is an intro science class question, you might just state that for the purposes of the problem solution, you assume the current flows at a constant rate, and that the fuse blows if more than 25 amps flows for a time approaching the 3 minutes. Then calculate the current (charge flowing per second) and compare it to the fuse rating. You will get a clear answer, given the assumptions. Edison (talk) 21:41, 24 September 2008 (UTC)[reply]
Is summer vacation over so soon? Plasticup T/C 23:38, 24 September 2008 (UTC)[reply]

Archie Frederick Collins

I am drafting have moved to main space a biography of this prolific science writer and early radio experimenter (claims to have made voice transmissions in 1899, was a business partner of Nathan Stubblefield) at User:Edison/Archie Frederick Collins Archie Frederick Collins. He used "A. Frederick Collins" as his name in publications. He wrote about 100 books on science, technology and hobbies, numerous encyclopedia articles and over 500 articles in technical magazines and newspapers from about 1900 to 1946. His books often bring handsome prices on Ebay and some are still in print. He wrote the first edition of "The Radio Amateur's Handbook" in 1922, still in print in its 82nd revised edition. He was born in Indiana January 8, 1869, but neither Who's Who (which carried a bio listing and a listing in "Who Was Who") nor the Library of Congress, in its bibliographic index, has a date of death. He seems to have died sometime after 1946, since that was the date of his last book. He was a resident of New York City and of Congers, New York. If anyone has access to New York state vital statistics to find a death certificate, or to any Rockland County, New York or New York City local history sources or probate records , or for an obituary which does not show up online, or any Florida sources (where he once had a residence) I would be grateful. A "real life reference librarian" could not turn up a death date in any bio compendium. His wife was Evelyn (or "Eva Lena") Bandy Collins and they had one son, also an author, Virgil Dewey Collins (born 1898). Thanks. Edison (talk) 21:28, 24 September 2008 (UTC)[reply]

Well, I have one teeny-tiny scrap for you:
http://www.daggy.name/cop/bkofdead/obits-co.htm narrows his death date to between 1949 and 1954 and says that he dies in Congers, NY.
The 1954 date is somewhat backed up by http://www.ibiblio.org/ccer/1926a1.htm which points out that the copyright of his book "A Bird's Eye View Of Invention" was renewed by Thomas Y Crowell Co on 20th Jan 1954. The annotation after the renewer's name is "(PWH)" which stands for "Proprietor of a Work made for Hire". There is another annotation for "Proprietor of a Posthumous Work" - so perhaps that is evidence that the author was still alive when his publisher renewed the copyright...but that's pretty thin evidence.
SteveBaker (talk) 23:38, 24 September 2008 (UTC)[reply]
The "Daggy" site seems to be guessing. The copyright renewal site might prove something, but it is hard to interpret. I do not see a "PPW" for Collins there, which would at least give a year by which he had died. I believe his publisher was absorbed by other companies, so their records may be unhelpful. I have looked for this information for several decades. The lack of a New York Times obituary for a frequent contributor to that paper suggests that the family wanted to keep it low-key. He was sort of the Mr. Wizard of the pre-television age, with many books on science for children and how to build neat things. Someone of his prominence could easily have had obituaries in papers such as the NY Times, but no joy there. A death certificate from the NY vital statistics office would be ideal. I really need a reliable source for a date of death so I can publish the article in mainspace. Edison (talk) 04:54, 25 September 2008 (UTC)[reply]
In the real of copyright renewals, it looks like here [5] and here [6] that A.F. Collins renewed (as author) the copyright of "The book of the microscope" on 7 August 1951, at the age of 82. Agreed? So his demise must have been after that date, if the record is accurate. Edison (talk) 05:18, 25 September 2008 (UTC)[reply]
Then 16 December 1955 there is the copyright renewal of a 1928 title "Boys' and Girls' Book of Indoor Games" renewed by Carolyn Collins Vilk, "next of kin," [7] , with the implication he had passed on or was incapacitated by that date. 23 Feb 1955 there is a copyright renewal [8] of "The book of puzzles", a 1927 book, by Myra K. Collins, listed as "executor of the author," so he seems to have died by then. Collins seems to have died circa 1951-1955. That would at least make a vital records search cheaper, since they charge by the time span. .Edison (talk) 05:38, 25 September 2008 (UTC)[reply]
Oh! Nice detective work! I didn't think to look for copyright renewals of his other books. But I strongly agree - "next of kin" certainly implies either death or at least a terminal condition. This is strong evidence for a range from 1951 to 1955. It's really bizarre that hundreds and hundreds of Google hits say "1869-" with no death date. Collins sounds like a really interesting guy though - I look forward to reading an article on him. Invented the mobile phone and then went to jail for fraudulantly advertising it - came out of jail and couldn't get a job so he writes dozens and dozens of truly excellent books...an interesting guy evidently. SteveBaker (talk) 23:47, 25 September 2008 (UTC)[reply]
I found several books on communications, and on popularized science, which had some discussion of the significance of Collins' research and writing. I have moved the article from my sandbox to main space. I still feel that the combined effort of Wikignomes should be able to find a more precise date of death for such a well known author than "circa 1951-1955." Edison (talk) 22:37, 26 September 2008 (UTC)[reply]

It would be great if a few experts in the fields of electronics, physics, Tesla's theories and inventions would look this article over. It certainly seems a most exciting issue, as the size of the article, its content, the strong opinions and some mentioned sources on the talkpage suggest that Tesla's concept of wireless energy transmission of industrial-level electric energy was and remains very feasible, but especially as they all suggest that Tesla obviously knew about stuff such as waves in plasmas, magnetohydrodynamics, the ionosphere, ELF transmission communication, intentional telluric current, Schumann resonances, planet earth's self-capacity and its use as a cavity resonator, and Zenneck waves as far back as around and shortly before 1900, and that his only problem was financial support as soon as his entrepeneur investors found out that he intended to provide free electricity out of thin air for everyone on earth as a quasi-socialist public service; J. P. Morgan asked Tesla, "Where do I put the meter?", but Tesla neither knew nor cared. --80.187.125.4 (talk) 21:31, 24 September 2008 (UTC)[reply]

Despite Tesla's major contributions to electrical knowledge such as advances in alternating current motors, robotics, radio, high frequency/high voltage alternating current, and high speed turbines, he had some scientific ideas that have not gained acceptance, or which have been disproved, and is often given credit for understanding issues which he may not in fact have followed, and for having reduced to practice things like "death rays" which were only wild talk in his old age for the benefit of credulous reporters. Edison (talk) 21:47, 24 September 2008 (UTC)[reply]
Edison, you're just jealous. Face it, you lost the "current war" - it's an AC world now. ;) Franamax (talk) 12:15, 25 September 2008 (UTC)[reply]
I guess I can refute you on at least one call: Tesla's diary entries from as early as 1899 document his understanding of the ionosphere, see Tesla, Nikola, "The True Wireless". Electrical Experimenter, May 1919. It was detailed enough for him to predict the correct ELF frequencies required, half a century before their practical military use, as based upon what's now called Schumann resonances. Note that the radiation Tesla was about to utilize at Wardenclyffe was not Hertzian in nature but in fact Zenneckian, as he was openly saying that he was dismissing Hertzian radiation for far-distance wireless energy transmission and going for Zenneck waves instead, and thus the waves would be propagating basically in plasma, in Tesla's case particularly using air, water, and the planet earth itself as natural media conductors (basically like solar wind plasma). Also remember he'd found a way to draw electric energy from thin air years ago in Colorado Springs already, which was actually the cosmic background radiation, as well as demonstrating wireless eletric energy transmission through the air at a level high enough to fully power head-sized lightbulbs that could be carried by hand hundreds of feet around without glowing any dimmer as early as 1891, as that's the year when he demonstrated it at the American Institute of Electrical Engineers as well as the National Electric Light Association and the Franklin Institute.
And calling his Teleforce design for a particle beam weapon a "death ray" makes it sound more ridicilous than both thes article particle beam weapon as well as teleforce obviously make his device out to be. The basic device you need for particle acceleration is a van de Graaff generator which had been around for half a decade when Tesla started pushing his teleforce design, and contemporary news coverage read that he was either to use a van de Graaff generator or an updated version of his magnifying transmitter. The teleforce obviously was close enough to production that he tried to sell it to several Western governments before WWII.
The other buzzwords mentioned above certainly weren't around by 1900 so it's no surprise that Tesla didn't use exactly these terms back then when explaining what his plant at Wardenclyffe was to be doing. Also, the Wardenclyffe plant certainly isn't much to do with his "later years", as he was still four decades from his death. --80.187.125.4 (talk) 02:14, 25 September 2008 (UTC)[reply]

How to store things on an earth floor yet keep them dry

I want to store boxes of books etc. in a small outbuilding which has an earth floor. I have done this before in the past, and I found that neither old carpets or sheets of plastic stopped the moisture from the earth seeping through and making the books damp and subject to insect attack. Can anyone suggest a cheap easy readily available thing to put between the earth floor and the books to keep them dry please? Or some other way of seperating them from the floor that supports them? Aluminium foil for example would be too fragile. Thanks. 78.147.10.10 (talk) 21:58, 24 September 2008 (UTC)[reply]

A wooden pallet or plywood supported by thich planks can keep the books up off the ground. If they are on the ground, even on plastic, the coolness of the ground could cause condensation. They still need protection from dust, leaky roof, rodent and bug infestation. A metal clothes cabinet up off the ground might help keep mice and bugs out. There are humidity absorbers available at hardware stores which have a chemical such as calcium chloride in them to absorb moisture. Some can be heated periodically by taking them and plugging them into an electric outlet elsewhere to get rid of the absorbed moisture. Sheets of old carpet or carpet underlayment (as thermal insulation) on top of plastic (as a moisture barrier) could also insulate from the coolness of the ground.Edison (talk) 22:12, 24 September 2008 (UTC)[reply]
You could store your books in one of those vacuum-sealed bags (I'm thinking of Space Bags...can you believe we have an article about them?!). These are huge polythene bags - you could store dozens of books in them - with a fitting that lets you hook up a vacuum cleaner and suck out most of the air. Alternatively, you could just stick each book into an individualised 'ziplock' baggie. The result should be pretty much resistant to everything...although if they're going to be there for a long time - you'd want to keep them out of sunlight. SteveBaker (talk) 23:07, 24 September 2008 (UTC)[reply]
A pallet would be great. You can sometimes get old or broken pallets for free from large retail stores or transport companies. As for the bugs use insecticide under the pallet. Mieciu K (talk) 23:39, 24 September 2008 (UTC)[reply]
I think the important thing is to get them up off the ground, wooden pallets as Edison suggests would be quite good. Once they're off the ground, you can just put them in bin bags. --Tango (talk) 23:47, 24 September 2008 (UTC)[reply]
Some plastic container, even a large plastic bin, would be good. Mice and bugs should be kept out, once the moisture problem is solved. Edison (talk) 04:49, 25 September 2008 (UTC)[reply]
Raising the books off the floor is key, so that there's air circulation underneath. When I had to store my books for several months, I put them in boxes (from the liquor store, which gives them away free) and bought 4-ounce bags of silica gel. They were rated as giving 3-month protection inside of the sealed unlined cardboard boxes. I would recommend the same approach here, except using hard plastic containers would be better to keep away the critters. Do throw in some desiccant though. Once your books are wrecked, they're wrecked forever. Franamax (talk) 12:05, 25 September 2008 (UTC)[reply]

Thanks, I do not think using pallets would be a good idea as the wood would get moist and thus be a magnet for various insects and rots (where I live the soil is moist - perhaps in desert areas in the US it is not). Plastic containers may work but I would need very many of them, which would be expensive. I was hoping someone might suggest some sheeting which is truly waterproof, as polythene is not. Some non-ferrous metal sheeting may be best, if I can get some cheap. 89.243.119.61 (talk) 19:56, 28 September 2008 (UTC)[reply]

You are almost dead-wrong here. The key is to allow air circulation between the earth and the books. We don't all live in the desert (or the US) - a skid (pallet) will work fine on dirt, that's what they do, but if you're worried, put the skid on four bricks so it doesn't touch the floor. Metal sheet will provide an equally good condensation surface, you need the air gap. However, if you are proposing to store books unprotected in an enclosed space with a damp-earth floor, you'd best give them away now. You will need all the factors - raised, in plastic boxes, with dessicant. Franamax (talk) 11:44, 30 September 2008 (UTC)[reply]

Both the wood of the pallet and its supporting bricks (unless engineering bricks were used) would gradually wick up moisture from the soil. It may be ok if supported off the soil by tin cans, for example. I do not see why a metal sheet would have more condensation that any other surface. Pond-liner may be a truly waterproof sheeting, but I think its quite expensive and not readily available. You are right that ventilation is needed. 78.149.137.49 (talk) 01:24, 1 October 2008 (UTC)[reply]

You're on the right track. Minimizing the contact surfaces is important, and airflow is important. The vertical edge of a pallet sitting on a brick will indeed wick some moisture but you are now dealing with only a few square/linear inches as opposed to putting down some carpet right on the dirt. Look at the moisture path - up a brick, to the bottom of the pallet vertical, up the vertical, across the surface of the pallet, into the books/boxes - that's a long path. My point about a metal sheet (or a pond-liner) is that any continuous surface will provide an avenue for moisture creep, the moisture will come out of the damp air and move along the surface.
Those waxy cardboard boxes would also provide a good sealed environment for the books (still should be kept off the ground though), and I'd still recommend a desiccant inside. Sorry for my passion on this subject, I just hate the thought of perfectly good books getting ruined! :) Franamax (talk) 01:41, 1 October 2008 (UTC)[reply]

DOes anybody know what's the colour of Pluto's sky. Could it be something else besides black like purple-black or dark blue. I thought it has thin atmospher.--57Freeways 22:37, 24 September 2008 (UTC)[reply]

According to Pluto, its atmosphere is more than a hundred thousand times thinner than Earth's atmosphere. I would expect the sky to be totally black to the human eye. Algebraist 22:41, 24 September 2008 (UTC)[reply]
And even if the atmosphere were denser - the sun is so amazingly dim at that distance that very little light would scatter from it - so yeah - VERY black! On the other hand, the lack of light pollution and the clarity of the atmosphere should make for astoundingly beautiful stars - and that HUGE moon...WOW! SteveBaker (talk) 23:00, 24 September 2008 (UTC)[reply]
Given the distance, I'd expect Charon to be pretty dark too, if not completely black.--Fangz (talk) 23:26, 24 September 2008 (UTC)[reply]
This artist doesn't think so! Wow. Plasticup T/C 23:34, 24 September 2008 (UTC)[reply]
Charon's albedo is about 3 times that of our moon and it's more than 10 times closer to Pluto than our moon is to us, so that will compensate a bit. --Tango (talk) 23:50, 24 September 2008 (UTC)[reply]
Being nice and shiny can only do so much. Pluto gets 0.06% as much sunlight as the Earth. Dragons flight (talk) 02:48, 25 September 2008 (UTC)[reply]
Hmmm - let's toss some numbers around: so Pluto orbits between 30 and 50 AU's from the Sun (the Earth orbits at 1AU). The sunlight will get dimmer as the square of the distance - so the sun appears to be between 900 and 2500 times dimmer on Pluto and Charon than it is here. If Charon is 3 times shinier - then it would look 300 to 800 times dimmer than our moon - although the lack of an atmosphere on Pluto would give that a bit of a boost. Being closer to Pluto won't really make it look brighter because it's not remotely like a "point source" like the sun is at that distance - it's 3 times smaller in diameter than our moon - but it's 10 times closer - so it's going to look about three times bigger than the moon looks from the Earth - that would be quite something to see...but yeah - it would be pretty dim even when it's a "full-moon" (er "full-Charon"?). Pluto and Charon are tidally locked - so if you could see Charon at all from your location on Pluto, it would stay put in the same place in the sky all day and all night. Then, just for fun, you'd have two other teeny-tiny moons (Nix and Hydra) to look at...they are really pretty tiny rocks though - as dim as they are, you might have a hard time spotting them with the naked eye from the surface of Pluto. SteveBaker (talk) 02:51, 25 September 2008 (UTC)[reply]
Good point about it being closer not making a difference - I was thinking about the total amount of light you would receive from the moon, rather than the brightness (which is defined as being per unit area). --Tango (talk) 15:07, 25 September 2008 (UTC)[reply]
This thread inspired me to do some reading about the planets and moons of the solar system last night. The view from Phobos sounds absolutely stunning. --Kurt Shaped Box (talk) 17:00, 25 September 2008 (UTC)[reply]
That is what I love about the Ref Desk. In answering questions and in reading replies, I am taken to parts of the encyclopedia that I might otherwise never visit. Plasticup T/C 21:53, 25 September 2008 (UTC)[reply]
Yes, indeed. Take a bunch of inquisitive people and the whole of human knowledge in an easy-to-read form and you get something kinda beautiful. I love the part that Phobos will soon (well, within 11 million years) break up due to tidal forces and Mars with have a spectacular (if short lived) low-altitude ring system...followed soon after by 1013 tonnes of rock raining down from the sky onto a line precisely around Mars' equator....WOW!....SciFi authors need to know this stuff - stories need to be written! SteveBaker (talk) 23:35, 25 September 2008 (UTC)[reply]
Of course, they'll need to change the 11 million years to 11 days, Phobos to our Moon and Mars to Earth, and then remove any resemblance to valid physics. They then need to blow up the Moon with a few nukes (ignoring the fact that the moon breaking up is the problem in the first place giving them extra credit on the valid physics thing) throw in an unrelated romance and it will be a box office hit, for sure! --Tango (talk) 23:54, 25 September 2008 (UTC)[reply]
Nice. I'd read an sf book in which Phobos gets crashed into Mars, but I didn't know it could happen naturally. Algebraist 23:58, 25 September 2008 (UTC)[reply]
Replying to Fangz, even on a moonless night without cloud cover, the earth is not pitch black (very dark true, but not totally black). Starlight and Zodiacal light surely would count for something, but I agree the artistic picture takes liberties with appearances (perhaps a long camera exposure rather than naked eye). See also Olbers' paradox for why not very bright from starfields. David Ruben Talk 19:48, 1 October 2008 (UTC)[reply]

How often does this dysfunction happen? How large can a Macropenis be? What are the possible implications other then the inability to have a normal sexual intercourse? Mieciu K (talk) 23:35, 24 September 2008 (UTC)[reply]

There is very little information about this in the literature. I think that use of this term is confined to boys who are undergoing puberty or who have not yet reached puberty. See human penis size. Axl ¤ [Talk] 12:09, 25 September 2008 (UTC)[reply]


September 25

Planets, homework for MY 5th grader, can't find in packet, please help!

This planet has rings and 15 moons.

This planet has 18 moons and 7 rings.

This planet has 8 moons, 2 that can be seen by telescope.

This planet is 93 million miles from the sun. (by ang)

this planet's orbit crosses Pluto every 248 years. —Preceding unsigned comment added by 75.132.46.189 (talk) 00:03, 25 September 2008 (UTC)[reply]

Uranus, Saturn, Neptune. Earth is 93 millions from sun, what you think? now Jupiter have at least 63 moon,s at least 60 for Saturn, at least 27 for Uranus, at least 13 for Neptune. Your book must be at least 10 years old.--57Freeways 00:14, 25 September 2008 (UTC)[reply]


So according to my daughter's book/packet: ( she's with grandma and i'm working) This planet has rings and 15 moons. JUPITOR

This planet has 18 moons and 7 rings. SATURN

This planet has 8 moons, 2 that can be seen by telescope. NEPTUNE

This planet is 93 million miles from the sun. (by ang) EARTH

this planet's orbit crosses Pluto every 248 years. EARTH

Yes, that was right 10 years ago if you look for a book copyright in 1996. But now scientist keeps finding more moons, and numbers of moons keep multiply. Jupiter, Saturn, Uranus, and neptune is a gas giant when you try to land you can't because there is no place to land. See the article about Jupiter.--57Freeways 00:51, 25 September 2008 (UTC)[reply]

Use this table to help.--57Freeways 00:57, 25 September 2008 (UTC)[reply]

Earth is definietely 93 million miles away from the sun.--57Freeways 00:58, 25 September 2008 (UTC) this planet's orbit crosses Pluto every 248 years. I'm pretty sure this is Neptune.Avnas Ishtaroth drop me a line 01:20, 25 September 2008 (UTC)[reply]

This is the Science desk - so it important to be 100% clear and scientific about this:
  • This planet has rings and 15 moons.
  • This planet has 18 moons and 7 rings.
  • This planet has 8 moons, 2 that can be seen by telescope.
There are no planets in the Solar System that fit any of those descriptions. I'm pretty sure none of them did even 10 years ago when this was printed. All three gas giants have uncountable numbers of rings and we'll still be finding more teeny-tiny moons 20 years from now! The number of moons that "can be seen by telescope" is a crazy statement. Where is the telescope? How the heck does the questioner think we found the other six moons if not by staring at them through a telescope?! Stupid, stupid, stupid. All three questions are beyond redemption. Even ten years ago, it was very apparent that we'd be finding more moons all over the place and that the question would likely be incorrect before the ink dried on the page. Worse still - it misses the beauty that is these ring systems. Jupiter has a donut as well as its rings. The hundreds of separate rings around Saturn are sometimes braided together and twist and un-twist as little 'nursemaid' moonlets orbit through the debris. It misses the point that moons and moonlets extend out from these large gas giants in uncountable numbers.
  • This planet is 93 million miles from the sun. (by ang)
I don't know what "by ang" means - but the only planet that is at approximately that distance is the Earth (which varies from 91.4 to 94.5 million miles from the sun - with an AVERAGE of 93 million miles). The "(by ang)" thing...WTF??
  • This planet's orbit crosses Pluto every 248 years.
Well - this is clearly INTENDED to elicit the answer "Neptune" - but it's not remotely correct. Pluto NEVER crosses Neptunes orbit - nor vice-versa! Not even close! It only looks that way on a typical top-down view of the solar system such as a teacher writing a textbook who knew less about the solar system than most of his students would see when struggling to write a quiz! If you look at the system in three dimensions (see picture to the right here) - you see that Pluto's orbit is so tilted that it never comes anywhere near Neptune. In fact, the closest that Pluto and Neptune EVER get to each other is about eleven times the distance from the Earth to the Sun! Pluto actually gets closer to Uranus than it ever gets to Neptune. But even if we're super-generous and we agree to use the classical "top down" textbook view - Neptune has an orbital period of 164 years and it "crosses" Pluto's orbit twice in each loop around the sun...so it crosses it TWICE in every 164 years - not ONCE in every 248. What they are thinking of is that Pluto's orbit is 248 years - but it too "crosses" the orbit of Neptune twice in each orbit - so even if the question asked which body crossed the orbit of Neptune every 248 years - they'd still be wrong. This question is SO far off being right...it's absolutely unbelievable. The orbital mechanics of Pluto and Neptune have been known for at least 60 years - so we can't even accept that the answer is a bit out of date...it's flat out WRONG.
So, this very, very sad, sorry excuse for a quiz is some how managing to teaching kids at least six or seven incorrect facts in the space of just FIVE questions - and the only one that's remotely right is confusing! Heck, I'm beginning to think one of those Creationist loonies could have done a better job of this! ("Which planet did God name after Donald Duck's dog?")
When people wonder why standards for science education is declining...this is the reason. Please tell your child's head-teacher that he/she should be ashamed to be pushing lies and half-truths onto the kids. It would be better not to teach them anything on that quiz than to teach them lies and half-truths.
SteveBaker (talk) 03:33, 25 September 2008 (UTC)[reply]
Steve, how disrespectful to those of faith. Everyone knows that Pluto is not a real planet. On top of which, Pluto was Mickey Mouse's pet!! Have you even read the bible? :) Franamax (talk) 14:44, 25 September 2008 (UTC) [reply]
Yes, I know - I was trying to cleverly cram as much disinformation into a single sentence as possible...kinda like that last question in the quiz. Not only is Pluto not a planet anymore and Pluto is indeed M.Mouse's pet, not D.Duck's. But also: The name came from an 11 year old girl called Venetia Burney, not "God". The "dwarf planet" is named for the Greek version of Hades. It was a full 10 years later that the cartoon dog was named after the planet, not the other way around. Therefore, the only appropriate answer to my question is, of course "Goofy"! (Also - I'm one of a decreasing number of people who actually sat down and read the Bible from beginning to end...it's nearly as funny as Goofy.)  :-P SteveBaker (talk) 23:20, 25 September 2008 (UTC) [reply]
Yes, all as told in 2 Disney 4:3-19, you have read it! :) I've read the whole thing too, twice actually. I particularly liked the sequel, "God: The NEW Testament" - it had a lot less sex (no "begat"s) but some really awesome special effects (Revelations). Franamax (talk) 02:54, 26 September 2008 (UTC)[reply]
All of that namby-pamby helping out the poor and needy thoughout the sequel is all very well - but I missed the whole killing off the youngest kids of an entire civilisation, raining blood (that stuff stains!), annihilating the entire ecosystem of the planet, screwing up people's brains so they all speak different languages...all great "vengeful god" stuff! SteveBaker (talk) 04:16, 26 September 2008 (UTC)[reply]
To be fair, Steve, there's at least a reasonable answer to the telescope bit. Many smaller moons were discovered via the Pioneer and Voyager missions, and were (at the time) quite invisible to Earth-bound telescopes. Given the HST and recent developments in adjusting for atmospheric interference, I doubt it holds any longer -- certainly the discovery of minor satellites around Pluto and such cast it into great doubt. But there's absolutely a way to discover a tiny moon without a telescope. — Lomn 15:54, 25 September 2008 (UTC)[reply]
Well, a lot depends on whether you consider the cameras on Pioneer and Voyager to be "cameras" or "telescopes". Wiktionary says "A monocular optical instrument possessing magnification for observing distant objects, especially in astronomy." and Telescope says "A telescope is an instrument designed for the observation of remote objects and the collection of electromagnetic radiation.". The 1500mm lens that Voyager used for it's narrow field camera certainly falls under both definitions. And with that much magnifications, it's definitely what I'd call "a telescope". HST had been in use for 6 years before those questions were printed and it can image even the two tiniest known moons of Pluto - those are only 60 km's or so across and 30 to 50 AU's away. If the supposed answer is "Neptune" then most of it's smaller moons are over 150km across so HST can certainly image them. SteveBaker (talk) 23:20, 25 September 2008 (UTC)[reply]
I read this as an ambiguous distinction also. Can be seen by telescope == depends on the telescope. Ground-based, space-based, set up in your back yard? Presumably the quiz is meant to be answered based on reading of a specific textbook - so the answers don't have to correspond to physical reality, they just have to satisfy the teacher that you've read, integrated and understood the supplied materials. That's really the basis of early education.
Followup question though: once the various moons and rings were discovered by exploration craft, haven't they been subsequently observed and tracked by ground-based instruments? Franamax (talk) 02:54, 26 September 2008 (UTC)[reply]
(Read, integrated and understood a lot of irrelevent and incorrect garbage from an outdated textbook...yes)
Probably, yes. The tricky part of spotting moons is that they are very small and dim and they move quite quickly. You need LONG exposure times to capture a picture of a very dim object. Once you know the moon's orbital parameters, you can program your ground-based telescope to track it for hours accumulating tiny amounts of light and after enough time you'll have a decent picture. But the snag is that you can't FIND a moon that way - if you don't know where it is and how it's moving, you just can't image it. So I think that's probably why the spacecraft-based discoveries were initially important. Another part of it is that accumulating light in ground based telescope like this is great for doing things like spectroscopic and redshift analysis - but it's not so good for seeing craters and interesting surface features - for that, you need a spacecraft nearby. SteveBaker (talk) 04:01, 26 September 2008 (UTC)[reply]
To a certain extent, the precise accuracy of information taught to a 5th-grader is not relevant, although it would be nice. Most people will get through life just fine with only a hazy understanding of the solar system - as long as they know planets orbit the sun, moons orbit planets, the whole thing is really big - knowing Pluto's orbital eccentricity doesn't help butter your toast. Learning how to learn is always the aim, and more precise facts can always wait 'til later schooling. And of course, now there's Wikipedia, where you can check to find out that the current number of known moons of Neptune is "my firend Janey is so awesome".
You need to also keep in mind the incredible cost of new textbooks and learning materials plied by various consultancies; also the fact that many elementary-school teachers by and large didn't learn any science themselves beyond fifth-grade. Franamax (talk) 06:37, 26 September 2008 (UTC)[reply]
I have to disagree with most of that. Things learned at an early age tend to stick very much. Why do you think oodles of unqualified (and indeed many qualified) people were interested in the status of Pluto? We don't need to teach all the details, of course, but we should make sure that we do not teach things that are plainly wrong. Yes, "learning to learn" is one aim. But such learning is much easier if you manage to get pupils interested in the material - and not disillusioned when they fact-check it somewhere else. And I dispute the "incredible cost of new textbooks and learning materials". For the price of one BGM-109 Tomahawk, we could buy about 10000-20000 elementary school textbooks. And unless my maths is wrong, you could bury each inhabitant (not every pupil!) of the US under 1000 textbooks per year calculating just the direct cost of the Iraq War. Of course, as a society we need to decide where our priorities are, but I would put more into the education of people and less into killing them. --Stephan Schulz (talk) 09:36, 26 September 2008 (UTC)[reply]
You are assuming the textbook is old. I've had the opportunity to review drafts of a middle school science text currently in preparation and it contained some ridiculous inaccuracies (on par with those above) even though it was being written now. The problem, as I see it, is that much of the work in preparing science texts for elementary and middle school education is being done by people whose primary background is either education or writing. Such people seem to assume that if they graduated high school they obviously know enough science to teach it to young people, and that blind faith leads to some ridiculous errors. Thankfully there are some scientifically trained reviewers, but all in all the state of science texts below at least the 9th grade level can be pretty abysmal. It's not uncommon to see old untruths regurgitated because that's what the writer learned when they were in school and they don't have the science background to know when knowledge has evolved. Dragons flight (talk) 09:53, 26 September 2008 (UTC)[reply]
It's instructive to read what Richard Feynman (my personal hero) thought about science textbooks - he took on the job of reviewing science books and was eventually defeated by the sheer frustrating task of fighting the system and trying to fix problems. I've tried (and failed) to get gross errors removed from several math and science textbooks in Texas. Even when the books were revised and reprinted - they ignored my error reports. In one case, a book has had the same error through three reprints despite three long and carefully reasoned/documented letters of complaint to the author, the publisher and the school authorities. I've had better success in getting my kids' grades fixed when he answered a question 100% correctly and was marked down because the official answer was hopelessly wrong. Some of the problems were pretty serious - for example, in geometry, it was exceedingly difficult to get the school authorities to agree that a square is a rectangle and a parallelogram and a rhombus...or that an equilateral triangle is an isosceles triangle and an acute triangle. Those are not small matters - the idea that because something is a member of a specific class, that this somehow removes it from a more general class is a severe academic error that can affect someone's thinking for years to come. Those kinds of problems go well beyond accidentally teaching that Uranus has the wrong number of moons. However, there is no excuse for having ANY errors in a textbook. SteveBaker (talk) 12:58, 26 September 2008 (UTC)[reply]

Btw, can someone tell me what a "packet" is, as used in this question title? Packet makes me think of cereal boxes, not textbooks (unless that is where the incorrect information comes from?) Gwinva (talk) 04:38, 26 September 2008 (UTC)[reply]

Sometimes, especially in grammar schools, teachers hand out material (photocopies of articles, worksheets, selected pages in books, etc.) in bundles or "packets" (often in manilla envelopes) and teach from that material rather than from textbooks. That's what's being alluded to here. Of course, it ought to be easier to keep such material up to date! - Nunh-huh 05:17, 26 September 2008 (UTC)[reply]
Thank you! I suspected that might be the case, but didn't want to assume. But "grammar school"? US usage = elementary school? Gwinva (talk) 07:49, 26 September 2008 (UTC)[reply]
Synonyms in U.S. usage. Probably older folk use "grammar school", younger folk, "elementary". Of course now it's all hopelessly confused, with "middle schools" and such :) - Nunh-huh 07:53, 26 September 2008 (UTC)[reply]
Again, thank you! Where I come from, grammar schools are secondary schools (high schools)... As for packets, they're generally just called "hand outs" or "work sheets". A people divided by a common language, and all that... Gwinva (talk) 07:59, 26 September 2008 (UTC)[reply]

wikitext needs your help

About 2500 words have been written in response to the OP. Maybe some of them could go into Wikijunior:Solar_System? Saintrain (talk) 16:40, 26 September 2008 (UTC)[reply]

Does anybody know color of Neptune's sky. To me it looks ocean blue. If Neptune's sky is blue how blue. Is it dark blue?--57Freeways 00:16, 25 September 2008 (UTC)[reply]

I'm not really sure how you define "sky" for a gas giant. There is no real surface to stand on and look up - by the time you reach anything solid it will be dark (well, there will be no sunlight - it's actually white hot at those depths, I believe). The atmosphere above you will most likely look very different at difference altitudes. The Earth's sky is blue due to Rayleigh scattering, Neptune looks blue from the outside due to its absorption spectrum - those are very difference causes. I think the absorption spectrum would still make the sky appear blue from the inside, but I don't know what would happen with scattering, so it could be any colour. --Tango (talk) 01:58, 25 September 2008 (UTC)[reply]
I think Tango's caveats still apply. "Above the cloud tops" is a fairly vague definition for a gas giant. Neptune's Great Dark Spot, for instance, is thought to be a gap in cloud cover rather than a cloud itself. Additionally, "true color" is a very tricky concept when it comes to photography. Note NASA's photo gallery of Neptune, where Voyager's near-true-color image is darker and less saturated than the HST's. Images at SolarViews show the same contrast. — Lomn 15:38, 25 September 2008 (UTC)[reply]
Above the clouds? Wouldn't that be above the main atmosphere? If it is then the "sky" would appear black. ~AH1(TCU) 01:41, 26 September 2008 (UTC)[reply]

But the article said neptune's sky is blue. Uranus is too. neptune I thought has a haze layers which would put the sky color to be ocean blue. It wouldnot be black because it have an atmosphere. Uranus have a haze layers so it's sky is thought to be light blue, just like Earth's same as Saturn's.--57Freeways 22:36, 26 September 2008 (UTC)[reply]

Those are photos taken from the outside. "Sky" is what you would see looking up from inside the atmosphere, that's not necessarily the same colour as the atmosphere from the outside. --Tango (talk) 23:03, 26 September 2008 (UTC)[reply]

Global Warming Counterargument

I was debating this subject with someone, and he said that the temperature will not continue to rise indefinitely as proposed by "An Inconvenient Truth" (I'm not sure that Al Gore was making that point; it's what my opponent said) and that if one examines the history of Earth's climate, we are in a relatively stable period; essentially, stability is abnormal. Is there any truth to these claims?CalamusFortis 04:29, 25 September 2008 (UTC)[reply]

Scientists have been arguing over this topic for years, so unfortunately we can't provide THE answer. For starters, you might enjoy "The Great Global Warming Hoax?", which predicts global cooling. Also New Zealand Climate Science Coalition, which offers links to many interesting research articles. Gwinva (talk) 04:49, 25 September 2008 (UTC)[reply]
I don't think anyone claims the temperature will rise indefinitely. If they do, they are clearly wrong. There is only so much heat coming from the sun, so the planet will have to reach equilibrium eventually, whatever happens. --Tango (talk) 14:46, 25 September 2008 (UTC)[reply]

You should also know that many of the scientists who were quoted on the great global warming hoax consequently sued C4 for taking their remarks out of context...82.22.4.63 (talk) 17:39, 25 September 2008 (UTC)[reply]

Did they? Nice to know. I was merely offering links which might provide some of the "counterarguments" requested by the OP, but since I've not read either of them, I'm certainly not in a position to judge their academic rigour. Gwinva (talk) 02:41, 26 September 2008 (UTC)[reply]
Indeed. And neither Al Gore nor any scientist worth the appellation claims or claimed that temperature will rise "indefinitely". As a first, very rough approximation, it cannot become hotter than the maximum of its current core temperature and the surface of the sun without violating the second law of thermodynamics. More realistically, the current predictions center around about 3° Celsius temperature increase per doubling of atmospheric CO2. The normal difference between icehouse and hothouse modes of Earth's climate are about 10° degrees Celsius (globally - much more in high latitudes), so I would not consider a very much higher rise due to anthropogenic greenhouse gases likely. Note, however, that even that would have devastating effect - including complete meltdown of the ice caps, sea level rise of 60-70m (enough to flood most costal plains and cities), and corresponding social and political unrest (to put it mildly). On the plus side, all that will take a long while - IPCC projections for 2100 are "only" 9-88cm, and while thy are conservative, they are not outrageously so. And yes, while civilization will suffer and may even break down, life will almost certainly go on for several million years more. --Stephan Schulz (talk) 18:10, 25 September 2008 (UTC)[reply]

Certainly an "indefinite" rise is right out of the question...however, it seems technically possible that we could conceivably drive the planet to the point where it looks like Venus...a surface temperature of 460 degrees C. (Well, actually, the earth is further from the sun - so it would be a bit lower than that - but not by much). To get to that point, we'd have to initially drive the temperatures up to the point where deep ocean methane-hydrides would melt and dump methane (a nasty greenhouse gas) into the atmosphere and the oceans. That would cause plants and algae to die off on vast numbers - that would start to shut off CO2 absorption - CO2 from rotting vegetation, forest fires and volcanoes would build up in the atmosphere causing still higher temperatures - more plant die-off - less CO2 absorption - more heat...with nothing to stop it. Gradually, the increasing acidity of the atmosphere combined with higher temperatures would cause carbonate rocks to start to boil off CO2 - yet more greenhouse gasses - eventually, we'd hit 100 degC and then the oceans would boil - and the resulting enormous quantities of water vapor (which is an even bigger greenhouse gas than CO2) would drive things even further off the rails. As these additional gasses enter the atmosphere, it gets denser and denser - and that too pushes the heat up and promotes other chemical reactions to acidify the atmosphere. Eventually, thermodynamics demands that things level off - but by then, Earth is Venus-2.0! So I think if we ever let things get so bad that large scale plant die-offs started to happen then we'd be unable to prevent runaway heating of the planet...reaching maybe 200 to 300 degC (Venus is at 460degC...so it's not impossible).

But it's pretty academic because we'd all be dead LONG before that happened.

It seems very unlikely to me that things could possibly get that bad - but there are a lot of 'runaway' situations here and when you get "positive feedback" in a system, it can be impossible to stop. It's hard to know for sure though because some things (such as increased evaporation of the oceans) can both help and hinder the process. More evaporation means more greenhouse-gasses (water vapor) - but it also means more fluffy white clouds which reflect sunlight back out into space again. In those cases, it's incredibly hard to predict when the point comes where the increase in cloud cover balances the increase in greenhouse gasses.

But I certainly wouldn't want to exclude the possibility of Venus-like conditions arising from this situation.

SteveBaker (talk) 22:43, 25 September 2008 (UTC)[reply]

Hi. Although stability is rare, on a human history scale we have been enjoying this stability for a very long time. So, if a sudden change occured, it would seriously disrupt civilization, becuase all of modern civilization has developed during this stability. Quoting another Wikipedian, such a drastic temperature rise like the one predicted "would be seen as nothing short of apocalyptic". Thanks. ~AH1(TCU) 01:39, 26 September 2008 (UTC)[reply]
Although we can only hope that SteveBaker's apocalyptic scenario won't arise, there is a potential trigger to release those oceanic clathrates. Permafrost areas in the Arctic have unusually large stores of unprocessed carbon, i.e. plant matter that has not yet been processed by microbes. Warming could potentially release this carbon, most unpleasantly as methane. I can't find the disturbing reading on the "budgets" involved (the potential releases would dwarf total anthropogenic emissions), but I did find a few papers [9] [10]. Of course, like everything GW-related, there is intense controversy on this. However, if we do get caught in a feedback loop (and considering that GW is manifesting strongly in the Arctic, it's not impossible), that little bit of extra warming could be enough to release methane hydrates from the ocean on a large scale. Then we would have a problem! Franamax (talk) 03:18, 26 September 2008 (UTC)[reply]
What makes the "runaway to Venus" unlikely is that the temperature of Earth was quite a bit higher at some times in the past (due, among others, to changes in orbital cycles, continental drift, and tectonic processes affecting the atmosphere), and that we quite possibly already survived at least one Methane hydrate gasification - well, the lucky 10% or so of species did. Of course, this may well be a stochastic process, and we got lucky. We may or may not get lucky the next time (if any), especially considering that the sun is (very slowly) warming up... --Stephan Schulz (talk) 19:26, 26 September 2008 (UTC)[reply]

Chemistry

What is the basic Difference between Radical and Ion.I Think that Radical is the Group of charge molecule. EG NH2+ please satisfy me.i will be very thankful to u. —Preceding unsigned comment added by 119.152.246.120 (talk) 05:24, 25 September 2008 (UTC)[reply]

A radical is a molecule, atom, or ion with an unpaired electron. An ion is a molecule or atom with a different number of protons than electrons. An ion may be a radical, but there are ions that are not radicals, and radicals that are not ions. Someguy1221 (talk) 05:39, 25 September 2008 (UTC)[reply]

Histogram of distribution of skin color

I had an odd thought last night: what would an accurate histogram showing the distribution of skin color in the world look like? Anybody have a guess, or the data needed to put one together? --98.217.8.46 (talk) 12:28, 25 September 2008 (UTC)[reply]

Von Luschan's chromatic scale has a map showing global distribution according to his classification, however it doesn't deal with respective populations of the area. -- MacAddct1984 (talk &#149; contribs) 14:02, 25 September 2008 (UTC)[reply]
Right, I'm interested in populations. But I'm not interested in imagined "native populations", either, I'm interested in the current population. --98.217.8.46 (talk) 14:48, 25 September 2008 (UTC)[reply]

2D PAGE not sensitive to low-abundance proteins

I'm reading a journal article about a 2D PAGE which did not show some of the proteins expected "possibly due to their low abundance". They then say "more comprehensive analyses will be performed to identify the altered low-abundance cellular factors by using large-size gel slabs, depletion high-abundance proteins, enrichment of the samples by pre-fractionation and by using silver staining coupled with LC-MS/MS analysis". How do you suppose they might deplete high-abundance proteins, leaving the low abundance ones unaffected? What do they mean by pre-fractionation? Is silver-staining more sensitive or does it just enable the LC-MS? --Seans Potato Business 13:21, 25 September 2008 (UTC)[reply]

Using high performance liquid chromatography and liquid chromatography-mass spectrometry, i.e. LC-MS. See this reference. Silver staining assists PAGE. See this reference. Axl ¤ [Talk] 16:15, 25 September 2008 (UTC)[reply]
I have an article on my desk I've intended to read for months now. "Recent Developments in High-Abundance Protein Removal Techniques" in LCGC North America vol 26 no 3 p 278 by Tim Wehr. My quick skim of the article suggests it is exactly what you are asking about. I think that articles from this non-peer review mag are available on line for free. ike9898 (talk) 16:42, 26 September 2008 (UTC)[reply]
Hey, thanks for the tip. A quote from the arrticle: Using "split-couple-recombine" combinatorial systhesis, a library of hexapeptides bound to macroporous 65-μm-diameter polymethacrylate microbeads is produced, with each bead carrying a unique peptide sequence. The sequential coupling of the 20 protein amino acids results in a library of 206 unique beads. These are packed in spin columns with a 100-μL bed volume. In contrast to antibody-based immunodepletion products, which typically have a capacity of 100 μL or less, sample volumes of 1 mL and more can be applied to the ProteoMiner beads. Because there are a limited number of binding sites for each protein, HAPs quickly reach the bead capacity, and excess HAPs are passed through the column. After sample introduction and washing, bound proteins are eluted from the column with a small volume (≤300 μL) of elution buffer (5% acetic acid, 8 M urea, 2% CHAPS). The net result is depletion of HAPs accompanied by at least a threefold increase in the concentration of LAPs - is this not a ridiculous idea? Surely some LAPs (low abundance proteins) simply wont bind to a hexapeptides and be flushed staight through? I see no reason to assume all or even most LAPs would recognise any hexapeptide. --Seans Potato Business 22:42, 27 September 2008 (UTC)[reply]

physical draw backs of jogging

1.does jogging makes a man slim quickly but as we leave the routine ,fatness reverses fast again. 2.does it have any drawbacks for jogging after some eating. 116.71.188.65 (talk) 14:26, 25 September 2008 (UTC)[reply]

Moved poster's sig to proper spot. Franamax (talk) 14:34, 25 September 2008 (UTC)[reply]
We can't give medical advice. Ask a doctor/nutritionist/personal trainer. --Tango (talk) 14:42, 25 September 2008 (UTC)[reply]
This isn't medical advice...1) Jogging, like any physical activity, will help to burn calories - that's not guaranteed to cause weight-loss but increased exercise is reasonably well correlated with becoming 'healthy' (so if you are overweight it may burn off fat). Obviously if you stop jogging then your calorie-burning will reduce (unless you replace the activity). To combat this you could take in less calories, or burn calories through other activities. 2) Jogging after eating might be uncomfortable if you feel 'full up' so aren't as flexible. Similarly running on an empty stomach could cause discomfort too. 194.221.133.226 (talk) 14:52, 25 September 2008 (UTC)[reply]
The first question might not be, but the second is. If we say there are no drawbacks and then the OP goes jogging after eating and falls down ill because of it, we could end up in a lot of trouble. --Tango (talk) 15:00, 25 September 2008 (UTC)[reply]
No, we wouldn't end up in trouble. The no medical advice bit is not a legal requirement. In general, there's no reason not to jog after eating, it is up to your own preference. Some people are fine with it, others feel uncomfortable. This is not medical advice tailored to your situation, this is just a general statement of pretty commonly known information. --98.217.8.46 (talk) 22:59, 25 September 2008 (UTC)[reply]
I believe the Wikipedia:Medical disclaimer prevents us from getting into legal trouble. We don't give medical advice here to protect OPs from harm. Zain Ebrahim (talk) 12:42, 27 September 2008 (UTC)[reply]
I agree with anonymous editor 194.221.133.226. This is not "medical advice". A doctor or nutritionist would be no better informed than anyone else (although a personal trainer might be). As 98.217.8.46 states, the answer to question 2 is derived from common sense and general knowledge. Axl ¤ [Talk] 07:39, 26 September 2008 (UTC)[reply]

How is water taken up by the body?

I feel silly for asking, but I seriously can't find any good info. I guess my search terms are pretty bad. When drinking water, where in the body is it taken up? Does stomach acid matter at all? Does having a full/empty stomach matter? And, perhaps most importantly, how long does it take to clear out of the stomach? Thanks in advance. -- Aeluwas (talk) 14:39, 25 September 2008 (UTC)[reply]

It's absorbed in the intestines, both small and large, according to those articles. I don't know about the rest of your questions. --Tango (talk) 14:58, 25 September 2008 (UTC)[reply]
From Guyton's Textbook of Medical Physiology, 11th edition, chapter 65. Very little water is absorbed in the stomach. This is because the internal lining (epithelium) lacks villi, which are abundant in the small intestine. Between six and eight litres of water are absorbed from the small intestine each day. A further one and a half litres are absorbed in the colon. [Most people drink between one and three litres each day. The "extra" water absorbed comes from secretions into the gastrointestinal tract.] Stomach acid makes no difference. Axl ¤ [Talk] 16:50, 25 September 2008 (UTC)[reply]
Thanks guys. I also found these links: gastric emptying and the effect of omeprazole on water gastric emptying. It looks like ~15 minutes would be a normal time to get rid of 50% of the water. -- Aeluwas (talk) 19:55, 25 September 2008 (UTC)[reply]

Hair Growth in Mammals

in many cows ,dogs etc the hair cant grow from certain point but coats ,man etc it grows continously .can some explain me this fact. —Preceding unsigned comment added by 119.154.2.133 (talk) 16:20, 25 September 2008 (UTC)[reply]

Hair follicle goes through cycles. At the end of the cycle the hair is shed. Some animals (cows) have shorter cycles so the hair stays short. Others (people) have longer cycles. A person's eyebrow's follicles have shorter cycles (a few months) than head hair follicles ( a few years) so head hair can grow much longer before it sheds but eyebrows stay short. Saintrain (talk) 19:23, 25 September 2008 (UTC)[reply]
But then why would human head hair have such long cycles? --Kjoonlee 23:46, 26 September 2008 (UTC)[reply]

Are cows immune to CS gas?

Just heard someone on a radio phone-in quote the factoid that cows are supposedly immune to the effects of CS gas. Anyone able to confirm/deny? It doesn't sound to me like something that anyone would even bother to test... --Kurt Shaped Box (talk) 16:57, 25 September 2008 (UTC)[reply]

CS gas does not address cows specifically, but says Notably, CS has a limited effect on animals due to "under-developed tear-ducts and protection by fur". -- Coneslayer (talk) 17:03, 25 September 2008 (UTC)[reply]
I'm sure they would bother to test it, since virtually all chemical weapons (even mild ones like this) are tested extensively on animals. --Sean 18:57, 25 September 2008 (UTC)[reply]
But I don't know if cows are a common test subject Nil Einne (talk) 20:33, 25 September 2008 (UTC)[reply]
I've been pondering this on-and-off since I posted the question. Perhaps a herd of cows was used at some point as an experimental substitute for a crowd of hostile rioters when testing CS gas grenades in the open air? Only thing I can think of (though it doesn't really sound plausible)... --Kurt Shaped Box (talk) 22:11, 25 September 2008 (UTC)[reply]
Is there a plausible situation in which the use of CS gas against protesters also affected a herd of cows? It could have been an incidental observation. Plasticup T/C 22:31, 25 September 2008 (UTC)[reply]
These sorts of factoids are often over-specific to make them seem more obscure. (ie : "It's illegal to hitch a pet [exotic animal] to a fire hydrant.") Perhaps someone took the fact that animals are resistant to CS gas and decide to restate it as "cows are resistant". It wouldn't take too many retelling for that to become "cows are immune". APL (talk) 01:07, 26 September 2008 (UTC)[reply]
Cows have big eyes as well as noses with mucous membranes, neither protected by fur. I doubt that a cow in a cloud of CS would be immune to its effects. As soon as the irritant contacts the eye, the interior of the nose, the mouth and the lungs it has its effect. Having fur might protect somewhat against direct skin contact, which is the least of your worries. Edison (talk)

Why do people prefer creamy skin lotions compared to solid or liquid formulas?

I got addicted watching skin cream ads on YouTube and wondered why every lotion and cream for the skin is a whipped white concoction? Is there something humans like about white creamy lotions as opposed to colored oily or watery formulas? Do any wild animals put anything on their skin to soothe dryness? --70.167.58.6 (talk) 22:19, 25 September 2008 (UTC)[reply]

Any coloring agent in the lotion is likely to stain skin and clothing. --Sean 22:54, 25 September 2008 (UTC)[reply]
Pigs roll in the mud to cool themselves off... I don't know if they get any moisturizing effect out of it to (or get one intentionally, anyway). --98.217.8.46 (talk) 23:03, 25 September 2008 (UTC)[reply]

With a creamy lotion, it's easier to control where you're applying it than if it's more watery. --Anonymous, 03:03 UTC, September 25, 2008.

What about it was gelatinous or paste? --70.167.58.6 (talk) 16:30, 26 September 2008 (UTC)[reply]
Well, people certainly use petroleum jelly (a clear jelly/paste) as a moisturizer. (The original Vaseline was petroleum jelly.) Although it works (sometimes better than other moisturizers), petroleum jelly is usually not preferred anymore because of the "greasy feel". If you listen to the adverts, you'll see they'll often mention "it doesn't feel greasy!" Also being noncomedogenic (not clogging pores/forming acne) is important for some people - petroleum jelly may cause acne issues, depending on how/where it's used. The greasy/non-greasy issue is also probably why medicinal creams are preferred these days to ointments and liniments -- 128.104.112.147 (talk) 19:41, 26 September 2008 (UTC)[reply]
So what makes "greasy" skin cream less undesirable? What's the psychology behind that? --70.167.58.6 (talk) 14:50, 29 September 2008 (UTC)[reply]
I don't know exactly, but I'd guess it's due to discomfort with there being "something" on your skin, or concern over transferring the grease elsewhere. Ointments/moisturizers are usually applied with the hands, and once finished applying, you usually have to clean up your hands with a "greasy" product (or else there is still the "grease" on your hands), while non-greasy products usually absorb into the skin well enough not to require cleanup. -- 128.104.112.147 (talk) 23:18, 30 September 2008 (UTC)[reply]

QFT Example?

The pages under quantum field theory seem to lack examples. Could someone show an example of, say, an electron in a box, perhaps radiating photons to go to lower states, using a QFT, so those of us who aren't intimately involved in it can appreciate better how it works? --Zemylat 22:44, 25 September 2008 (UTC)[reply]


September 26

"hurricane" in France

Hi. I think I read somewhere that a "hurricane", most likely an extratropical cyclone, struck France causing $88 billion in damage sometime in the late 20th century, second costliest worldwide only to Katrina. Also, is there an article on this? The global tropical cyclones track image, a featured picture, which does not distinguish between extratropical and tropical systems, shows at least one category one hurricane approaching France. Also, is there an article for the hurricane-strength cyclones that struck Britain I think in 1990 (and killed Emily Macdonald), and recieved "names"? Also, does anyone know is there an online database where I can search, for example, the last time (year, name) that a tropical cyclone of x intensity approached within x km of a specified location, etc? Thanks. ~AH1(TCU) 01:54, 26 September 2008 (UTC)[reply]

Wikipedia has a pretty extensive catalog of tropical cyclones:
-- MacAddct1984 (talk &#149; contribs) 02:17, 26 September 2008 (UTC)[reply]
The French one was in December 99, just 1 or 2 days after Christmas. I cannot remember the name of the cyclone, though, sorry --Lgriot (talk) 02:28, 26 September 2008 (UTC)[reply]
I found it!: "The second storm event occurred on December 26. It was caused by a low named "Lothar", and left a trail of destruction from northwestern France to southern Germany and Switzerland." full text at: [11] . We also have an article: Lothar_(Storm) --Lgriot (talk) 02:37, 26 September 2008 (UTC)[reply]
Hurricanes have to be tropical cyclones (or sub-tropical cyclones, as the definition was expanded recently). Extratropical cyclones, not matter how strong, cannot be hurricanes. See Cyclone, specifically Extratropical cyclone, Subtropical cyclone, and Tropical cyclone. Plasticup T/C 03:35, 26 September 2008 (UTC)[reply]
Following the external links in the Lothar article, I find that it caused 5.0 billion

Euros damage - less than the 1990 storms which hit France. So the "second costliest worldwide only to Katrina" part cannot be correct. Hurricane Katrina caused about $100 billion damage. Rmhermen (talk) 13:25, 27 September 2008 (UTC)[reply]

Hi. Yes, but farther down in the pdf it states that total damage was over 100 billion euros. Also, is there an article for the 1990 storms? What about the database? Thanks. ~AH1(TCU) 19:54, 27 September 2008 (UTC)[reply]

ACCA

Hello I want the free Notes for ACCA if some help me i will be very thenkfulPDF). Also what is the salary given to ACCA with one year experience in UK(in digits please) —Preceding unsigned comment added by 119.154.29.138 (talk) 02:39, 26 September 2008 (UTC)[reply]

Association of Chartered Certified Accountants and Chartered Certified Accountant may be of some help. SteveBaker (talk) 03:42, 26 September 2008 (UTC)[reply]

Kinetic Energy of Helium

Yup, I'm doing homework. I thought I'd done the question right, but can't get my online homework board to accept my answer. To find the total kinetic energy of 2.4 kg He at 298.15 K, I used the formula KE (avg) = (3/2)RT, then multiplied this by the number of moles.
So: (3/2)(8.314 J/mol*K)(298.15K) = 3718.2 J/mol
(2.4 kg He)(1000g He/1 kg He)(1 mol He/4.003 g He) = 599.6 mol He
(599.6 mol He)(3718.2 J/mol He) = 2.229E6 J

There is a second part to this problem, but I thought I'd see if I was doing something incorrectly in this first part, or if I am at least right to this point? Any pointers to my approach and setup of this problem would be appreciated. 166.70.30.42 (talk) 07:19, 26 September 2008 (UTC)Andy[reply]
Nevermind...I figured it out. Damned significant figures get me every time... 166.70.30.42 (talk) 07:40, 26 September 2008 (UTC)Andy[reply]

bird nomenclature / indie rock lyric question

Joanna Newsom is a (really hot) singer who writes rambling 11-minute songs bursting with precious avian references. One memorable opus begins, "The meadowlark and the chim-choo-ree and the sparrow set to the sky in a flying spree, for the sport over the pharaoh". What the heck is a chim-choo-ree? I searched the encyclopedia for a bird by that name but found only the Mary Poppins song.--The Fat Man Who Never Came Back (talk) 07:52, 26 September 2008 (UTC)[reply]

ArthurMag has an article on Joanna Newsom where it is suggested that she probably derived "chim-choo-ree" from Mary Poppins. --Bruce1eetalk 14:13, 26 September 2008 (UTC)[reply]
Thanks. That's a good reference--funny too: "Meadowlarks and sparrows are songbirds, of course, but the chim-choo-ree, as far as I can tell, is probably an outgrowth of watching Mary Poppins too many times." But isn't the song "Chim Chim Cher-ee" about chimneys and not about birds? Ms. Newsom is an odd duck. Anyway, sorry to bother the Science desk with this pop culture rubbish; I imagined that a chim-choo-ree was some sort of archaic slang for a specific songbird.--The Fat Man Who Never Came Back (talk) 16:03, 26 September 2008 (UTC)[reply]

Severity of Decelleration Required to Cause Traumatic Brain Injury

I've seen that rapid decceleration can cause diffuse brain injury in numerous places, and that this can occur/ is likely to occur in auto accidents. However, I have never seen it mentioned exactly how severe the accident needs to be, nor the neccesary ammount of decceleration. So, this is my question: what level of acceleration is neccesary to cause brain injury, is coming to a quick stop without impact in a motor vehicle capable of causing brain injury, and (pending upon the answers to the first two) how is it that proffessional race car drivers/astronauts/sky divers/etc. do not sustain injury for these reasons? Thank you in advance for any help. Phoenix1177 (talk) 13:35, 26 September 2008 (UTC)[reply]

Hmm, well internal bleeding is common from deceleration in high speed accidents. Since this can injure your organs, this would presumably apply to your brain as well. I would assume it doesn't affect (or is it effect?) the people you listed because they don't actually decelerate all that quickly. If a professional driver were to slam into a wall and go from 150mph to 0 in no seconds flat, internal bleeding would probably be a major concern (or the least of their worries?). -- MacAddct1984 (talk &#149; contribs) 13:54, 26 September 2008 (UTC)[reply]
Thank you for your reply. A large part of my question stems from the fact that someone wrecking into a wall at 60 mph has a moderate chance of brain injury, whilst it doesn't seem common sensical that someone going 60 mph and slamming on their brakes would have equal chances(mainly because people slam on their brakes a good deal and don't seem to sustain such injury) So, I figure that either those extra few miliseconds of stopping time and meters of distance with the brakes are a mitigating factor, or that most people involved in high speed action should be at high risk for brain injury. [the case for sky divers is probably unreasonable, but with race car drivers it would seem somewhat more logical] Phoenix1177 (talk) 14:07, 26 September 2008 (UTC)[reply]
If you're not in a race car, your braking and turning maneuvers won't be much higher than 1g. -- Coneslayer (talk) 14:27, 26 September 2008 (UTC)[reply]
If you hit the brakes full on at any speed the reduction in speed will be not even in the same league as a crash. If i'm correct I think a big thing is the 'g-force' you go under in a crash. There is G-force#Human_tolerance_to_g-force that may be of interest. In particular the 'involuntary' section showing crashes. A vehicle braking will get some g but it won't be anywhere near that of a crash. The more 'time' that can be introduced into the deccelaration the better, that's the purpose of crumple zones and airbags and etc. etc. 194.221.133.226 (talk) 14:21, 26 September 2008 (UTC)[reply]
This study demonstrated deformation of brain tissue during deceleration of 20–30 m/s2, with the neck unprotected. Allowing the neck to pivot freely increases the strain just above the base of the brain where the brain is mostly strongly adherent to the surrounding bone. Professional racing drivers and astronauts wear neck protection to prevent whiplash injuries. This also helps to reduce torsion of the brain at its base. Axl ¤ [Talk] 14:34, 26 September 2008 (UTC)[reply]
More specifically, a basilar skull fracture is the danger when your body stops faster than your head does. The bone sutures can split and tear open the sacs holding in your brain's precious bodily fluids. --Sean 16:54, 26 September 2008 (UTC)[reply]
You also have to worry about your head hitting the windshield/steering wheel and your chest hitting the steering column (wear your seatbelts!). Although, I did go on a call to a car accident where the patient had a collapsed lung, even though the airbag deployed... -- MacAddct1984 (talk &#149; contribs) 17:45, 26 September 2008 (UTC)[reply]

Plant identification questions (yew and juniper)

I would like some help identifying some plants. I have some yew shrubbery and a juniper tree in my yard. I would like to know more specifically what type of yew and juniper I have. I live in Richmond, Virginia. The juniper is about 15 feet tall, and in a general cone shape. My brother believes it is Juniperus communis, but we weren't entirely sure how that differed from Juniperus virginiana. My concern is the berries. The berries are not round, but instead are sort of spikey. What sort of things can I look at to identify the type of juniper? Any details I could provide online that could help someone identify it? As for the yew, my brother believes it is a Taxus cuspidata, but I wasn't sure how that differed from Taxus baccata. Again, what sort of things should I look for to tell what type of yew it is?

I could provide photos if necessary, but I'd also like to learn what to look for myself. Thanks!-Andrew c [talk] 16:18, 26 September 2008 (UTC)[reply]

Evolution of axon membranes

We were studying the membranes of nerve axons today in biology and it confused me as to how such a system could evolve? We were studying different channel proteins and carrier proteins etc. Anyway, wouldn't any organism without such pumps like sodium-potassium pumps die? How else could organisms without proteins to carry out active transport survive? It seems unlikely that an entire membrane could evolve in one generation of a cell. I'm not trying to encourage creationism or anything, I do believe in evolution, it's just a bit confusing.

As a second kind of follow on question, are there any organisms that survive or is it at all possible for an organism to form without adenosine triphosphate mechanisms? Thanks a lot :)

Cyclonenim (talk · contribs · email) 17:18, 26 September 2008 (UTC)[reply]

"It seems unlikely that an entire membrane could evolve in one generation of a cell" - that's correct and as sure as I am that my username is Seans_Potato_Business, I don't believe it evolved over the generation of a single cell. There's not telling how each of the proteins involved (you can look for clues by checking sequence homology among genes and among organisms) but each gene and gene-product may have started off as something else. It's nor necessary (nor possible?) to known or understand each evolutionary step which occured. ----Seans Potato Business 17:36, 26 September 2008 (UTC)[reply]
I do appreciate that, it's not really a simple discussion. My point was, however, that wouldn't an organism die without just one type of these intrinsic proteins? If it didn't evolve all at once, how could it come into existence at all? —Cyclonenim (talk · contribs · email) 18:10, 26 September 2008 (UTC)[reply]
All organisms require ATP or GTP via biochemical respiration. However this paper suggests that bacterial motility may occur through a different intermediate in oxidative phosphorylation. Axl ¤ [Talk] 18:11, 26 September 2008 (UTC)[reply]
A modern organism might die without one protein from the pool. However this is no reason to suppose that an evolutionary ancestor would die without that protein. Axl ¤ [Talk] 18:14, 26 September 2008 (UTC)[reply]
It is noted that a wide range of living things have similar needs when it comes intracellular K and Na levels. As you are apparently are aware, much modern life maintains the right ranges by actively pumping Na and K against the natural gradient. The easiest explanation for how that situation might have evolved is if there was some very primitive progenitor that lived in an environment that didn't need pumping, i.e. to assume that once upon a time the external Na and K levels matched the internal levels we have today. The evolution of active pumping would allow a cell to escape that environment, but the existence of active pumping wouldn't be necessary to live there. Though I don't think it tells the whole story, the chemistry of the oceans themselves were radically different early in life's history (high Fe(II), lower Na, negligible O), so it certainly plausible that life initially began under very different environmental constraints than exist today. Dragons flight (talk) 18:26, 26 September 2008 (UTC)[reply]
Another way to look at it is this: It is possible that a proto-organism lived at equilibrium with the salt water environment, but that evolution of channels capable of separating charge across the membrane would be extraordinarily beneficial and allow the descendents of that proto-organism to thrive. The neuronal axon is an extremely specialized situation that has evolved over many millions of years. However, other cell types in the body also use gradients for transporting all sorts of things across the membrane, which you might imagine would be advantageous to a single celled organism in the primordial soup. Also, the ability to build up charge on the surface and discharge it upon some stimulus could be considered as a possible form of primordial warfare. There are plenty of explanations for how a complex adaptation like the neuronal cell membrane could evolve in small steps, each of which could be advantageous to the given organism in its particular context, without calling forth the "irreducible complexity" argument (i.e. "It seems unlikely that an entire membrane could evolve in one generation of a cell.") favored by the intelligent design crowd. By the way, you don't need to "believe" in evolution. You just have to understand how it is actually theorized to work, think up some testable hypotheses, and look for ways to confirm or refute them! Have fun. Medical geneticist (talk) 22:45, 26 September 2008 (UTC)[reply]

Making gold

Is it theoretically possible to make gold out of common atoms like hydrogen or carbon? Is there any serious scientific investigation going on in this field?Mr.K. (talk) 18:27, 26 September 2008 (UTC)[reply]

See Synthesis of noble metals#Gold -- MacAddct1984 (talk &#149; contribs) 18:35, 26 September 2008 (UTC)[reply]
Theoretically all heavy elements are derived from lighter elements (see Supernova nucleosynthesis). I'm not sure if something as light as hydrogen or carbon could be made into something as heavy as gold in a meaningful way with current technology (keep in mind the links above are just about going from specifically Mercury to Gold, which is just a proton difference). But theoretically it's possible—esp. if you were using fusion technology. But it's not likely ever a worthwhile replacement to mining. --98.217.8.46 (talk) 22:13, 26 September 2008 (UTC)[reply]
I can't help but wonder: what worth would gold have if you could make it in your cellar? hydnjo talk 23:22, 26 September 2008 (UTC)[reply]
It would still be useful for electrical contacts and it would still look pretty so would be used for cheap jewellery, so it would have some value. It would lose most of its value, though. --Tango (talk) 23:30, 26 September 2008 (UTC)[reply]
For sure gold has practical value. I was just musing about the non-practical value should gold become abundantly available as was suggested by Mr.K. There would be some advantage I'm sure in using gold foil in place of aluminum foil for some everyday applications ;) hydnjo talk 01:36, 27 September 2008 (UTC)[reply]
Beisides that, hydnjo, if I could make in my cellar that would not disturb the market price of gold. It would only improve my financial situation. Mr.K. (talk) 10:54, 27 September 2008 (UTC)[reply]
C'mon now Mr.K., surely you'd share your recipe with us and you know what blabber mouths we are. -hydnjo talk 12:33, 27 September 2008 (UTC)[reply]
The amusing thing is, Mr. K, if you had the technology to make gold out of carbon, you'd make more money licensing that than you'd ever make by producing gold. --98.217.8.46 (talk) 14:34, 27 September 2008 (UTC)[reply]
By selling the gold you make in your cellar you would depress the price of gold - it's simple supply and demand. If you were disciplined and only made a small amount of gold, the affect would be negligible, but if you tried to make massive amounts (something in the region of hundreds of tonnes, I'd expect) you could cause a massive crash in the price of gold. --Tango (talk) 15:22, 27 September 2008 (UTC)[reply]
I think he means if everyone could make gold in their cellars. — DanielLC 16:40, 27 September 2008 (UTC)[reply]
Also, that scenario reminds me of an episode of The Twilight Zone (specifically, the end). — DanielLC 16:42, 27 September 2008 (UTC)[reply]
Unless your particle accelerator gets free electricity, the energy input would cost orders of magnitude more than the gold would be worth. Cheaper, but still not cost effective, to extract the gold dissolved in the ocean. More cost effective to rvcover gold from electrical contacts in trashed electronic equipment. Edison (talk) 19:33, 29 September 2008 (UTC)[reply]

Extremely deadly tranquillisers?

I was watching The Lost World and one of the characters mentioned that if someone accidentally shot themselves in the foot with one of their tranqs, it would kill them instantly. Are there any real tranqs that are this deadly, where a single shot to the foot would instantly kill a man? ScienceApe (talk) 20:04, 26 September 2008 (UTC)[reply]

Tranquillisers don't take effect instantly. They take seconds or minutes to take effect. The dose of tranquilliser determines the risk of death. If the tranquilliser is intended to sedate an elephant, it could well kill a person. Axl ¤ [Talk] 20:22, 26 September 2008 (UTC)[reply]
Well in that movie it wasn't actually a tranquilizer, it was a refined neuro toxin derived from the venom of a conch shell cone snail, but I don't know how realistic this is (In fact I doubt it, I don't see how a shot in the foot could kill you that quickly before it has time to reach your heart, lungs, or brain). -- Mad031683 (talk) 16:45, 29 September 2008 (UTC)[reply]
It was apparently the cone snail, not the conch shell. My memory of the book describes a woman being stung by one, taking a few steps and falling over dead. The article seems to support a fairly rapid death from the sting of a large cone snail, but nowhere near as rapid as described in the book/movie. I'd buy death in a few seconds(as soon as the toxin reached the brain) but not so fast you don't even realize you shot yourself. -- Mad031683 (talk) 16:57, 29 September 2008 (UTC)[reply]

Tango, so you siad the sky color is not always the atmosphere on the top?? Well, Venus sky is orange-scarlet or coral seen from the surface because the orange-rd smog. Wouldn't the middle of cloud layers be yellow or yellow-orange. Does anybody know what's the color of Venus' sky above the cloud-tops, because Venus have 3 main layers of clouds.--57Freeways 23:20, 26 September 2008 (UTC)[reply]

A good example of a planet with a different coloured sky from the inside than the atmosphere appears from the outside is Earth - the sky is blue but seen from space the atmosphere is pretty much colourless. I don't know about Venus, I'm afraid. --Tango (talk) 23:27, 26 September 2008 (UTC)[reply]
  • Above the clouds is probably above the main atmospher of Venus. If so, then it's probably black. It's uppermost layers is probably acid clouds. Our sky is light blue over Trophospher', the next layer of atmos' is probably dark blue, the toppest layer of our sky might be indigo.--57Freeways 00:03, 27 September 2008 (UTC)[reply]

September 27

Where does my power come from?

I know it comes from a power plant. I live in West Virginia, US. We have a coal fired power plant less than 50 miles from where I live. Someone told me that the energy produced there is sent the the western US and we get our power from somewhere else. Is there anyway to figure out what specific power plant provides the energy for my area? —Preceding unsigned comment added by 96.240.72.130 (talk) 01:35, 27 September 2008 (UTC)[reply]

See Electric power transmission. As I understand it, the power generated at your local station is put into "the grid," which can be seen as a collective pool that consumers draw power from. So you don't actually get power from any particular power plant. It's possible that the electricity produced at your coal plant is being sold to cities in the West, and that your town is buying power from some other source, but that doesn't mean these plants are literally shipping their electricity to the buyers. It's all just going onto the grid. --Allen (talk) 03:09, 27 September 2008 (UTC)[reply]
And looking at the US grid map, WV is all in the East Central region of the Eastern Interconnect, so it is indeed the same grid. It is quite possible that the wires coming from the coal plant run in a westerly direction, but it seems unlikely that the power is being marketed into the Western Interconnect. Franamax (talk) 16:11, 27 September 2008 (UTC)[reply]
The power systems analyze flows on the grid by instruments located at substations, generating stations and on trnsmission lines. The amount flowing from each source and to each load varies depending on contidions. Computer modelling can determine the effect of adding more or less power to the system at any one node. If the effect of turning your local power plant on or off were compared to the flow to a load in the western US, I expect that lettle effect would be seen, In other words, it would be more likely that your local plant is not literally supplying western load. For one thing, there would be considerable losses of power along the way due to resistance. Edison (talk) 20:04, 27 September 2008 (UTC)[reply]

would a free falling object hit with the same force from 40' vs. 20'

my sons cubs scouts is having a experiment with an egg wrapped in a package. it will be dropped from 40' to see if it breaks. My egg was ok at 20' but i have no way of testing it from 40'. I was just wondering if it would hit with the same force at 20' as it would 40'209.244.187.94 (talk) 04:17, 27 September 2008 (UTC) Thank you Ray B.[reply]

Depends on the package. See Terminal velocity. If the package has a design that maximizes air resistance (like a parachute, for example), then it might hit with the same force at 20' or 40'. Otherwise, it will hit with more force. You might try testing by standing at 20' and throwing the package downward, trying to make it leave your hand at roughly the same speed it hits the ground at when dropped from 20'. --Allen (talk) 04:26, 27 September 2008 (UTC)[reply]
Just throw the egg up 20' from where you were standing before and let it fall from there. — DanielLC 16:21, 27 September 2008 (UTC)[reply]
That would have no effective difference from droping it at 20'. If the egg is at the top of the throw at 20', it is at zero velocity. From there, the downward flights are identical. The idea behind the egg drop is to make something that will reach a relatively slow terminal velocity as soon as possible. Thus, if it hits that terminal velocity at 15', it will drop at that same speed whether you drop it at 20', 50', or 1000'. That's the idea behind terminal velocity. --Jayron32.talk.contribs 20:40, 27 September 2008 (UTC)[reply]
I think you misunderstand. I believe DanielLC's plan was to stand 20' above the ground and then throw the egg 20' in the air, so it is momentarily at rest 40' above the ground, so it will follow the same path as if it were dropped from 40'. --Tango (talk) 20:51, 27 September 2008 (UTC)[reply]

Sounds like a cool experiment. What sort of package did you and your kid design? Plasticup T/C 05:05, 28 September 2008 (UTC)[reply]

The pedantic answer is that terminal velocity is asymptotic. So the egg will not ever hit at exactly the same force.--Fangz (talk) 12:53, 28 September 2008 (UTC)[reply]
I think random fluctuations probably have more of an effect than it not having quite reached terminal velocity. --Tango (talk) 20:57, 28 September 2008 (UTC)[reply]

Penrose Diagram

I have no idea how to interpret this. Could someone please help me out? Also, what is a "lightlike infinity"?

Ζρς ι'β' ¡hábleme! 05:57, 27 September 2008 (UTC)[reply]

I resized the image, hope you don't mind. It was a bit... overwhelming before. Anyway, this diagram just shows ordinary flat spacetime in an unusual coordinate system given by tan (u±v) = x±t, or more explicitly by
u = (tan−1 (t+x) − tan−1 (t−x)) / 2
v = (tan−1 (t+x) + tan−1 (t−x)) / 2
The axes marked "space" and "time" are actually u and v respectively. This coordinate transformation maps the whole of spacetime into the diamond. The roughly horizontal and vertical curves are curves of constant t and x. Lines of constant x±t map to lines of constant u±v, so light still travels along 45° diagonal lines in this diagram. Its most useful property is that worldlines which diverge to infinity in the usual coordinates converge to the boundary of the diamond in (u,v) coordinates, so this diagram shows you the mathematical structure of spacetime at infinity. Timelike geodesics extend from the bottom point of the diamond (called "past timelike infinity") to the point marked "timelike infinity" (normally "future timelike infinity"). Lightlike/null worldlines extend from past to future lightlike/null infinity (lower and upper edges of the diamond). You can extend this to n+1 dimensions by using polar coordinates and transforming (t,r) as above while leaving the angles alone. The diamond generalizes to two (hyper)cones with apexes at past and future timelike infinity (which are still points) and glued together at spacelike infinity (which is a (hyper)sphere). (edit to add: Sorry, I think that's wrong; that mapping isn't conformal. The correct generalization, if I'm not still confused, is to imagine wrapping the diamond around a cylinder so that the left and right vertices (spatial infinity) coincide, roughly as shown here, and then generalize that to a higher-dimensional cylinder Sn × R. Spatial infinity is a point, not a hypersphere.) -- BenRG (talk) 12:00, 27 September 2008 (UTC)[reply]
That sounds good. They're like those Escher Circle Limit pictures but putting the whole of a 2D space-time in a small diagram. I must admit I always though Penrose diagrams was the name of a much more useful thing which in wiki is called Penrose graphical notation. Dmcq (talk) 12:11, 27 September 2008 (UTC)[reply]
Yes, Escher's Circle Limit pictures are based on the Poincaré disk model of hyperbolic space, which is a very similar idea. Also very similar is the Riemann sphere. They're all conformal maps. -- BenRG (talk) 15:16, 27 September 2008 (UTC)[reply]

Changes in mental processes due to multitasking

I posted this a couple of days ago on the Misc desk as sex differences in internet use and it was suggested I post here. I rewrite slightly:

It is a commonplace observation that women are able to multitask and men cannot. (I should clarify that "multitasking" can involve more than paying attention to electronic information. A sympathetic female journalist was commenting on the recent resignation of Ruth Kelly from the British Cabinet, "to spend more time with her family". The journalist said that once she had interviewed Kelly by phone, and after a few minutes had figured out -- and the politician had admitted -- that the splashing sounds in the background were of the two year old being bathed. Good thing the phone didn't drop in the water.)

It is a commonplace observation that those individuals most immersed in computers and the internet are male. Obviously these are both generalisations. One social change commented upon over the last ten years or so is the impact of prolonged and submersive internet use on our thinking and interaction patterns. (See for example the recent Financial Times anecdotal article and interviews here.) One theme is that our attention is being fractured; some observers see this as wholy negative, while others see more positive possibilities.

So we have two conflicting ideas: that the so-called male brain can only cope with one information stream at a time, and that many men and boys thrive in an atmosphere of information bombardment (multiple screens on computers, phones, TVs simultaneously). My question is, is there any serious research on this? And how are these conflicting tendencies reconciled? BrainyBabe (talk) 12:14, 27 September 2008 (UTC)[reply]

Whatever about commonplace observance the latest statistics as best as such things can be done for this say that more women than men use the internet and use it if anything a bit longer. Just google on 'percenage male female internet usage' and look for 2007 or later. As to multitasking it is something to be avoided if possible even for simple tasks with little overlap if you want to do them properly. Anybody driving round talking on their mobile is a menace. Dmcq (talk) 13:02, 27 September 2008 (UTC)[reply]
Or driving around while fixing their makeup. :) Zain Ebrahim (talk) 13:04, 27 September 2008 (UTC)[reply]
Yes, I think it's the assumption that multitasking is an amazing skill that is the problem with the whole thing. People assume that being able to do 5 things at once means you get 5 times as many thing done (and done just as well). You don't. Totally my own opinion here, but I suspect an evolutionary reasoning for the multitasking/singletasking divide. Men were hunters. Attempting to hunt a wild animal while also reading the paper=dead. Remaining focused on the task without distraction=alive. Fribbler (talk) 13:48, 27 September 2008 (UTC)[reply]

Thank you for your opinions, but can anyone point me to any research on the subjects under discussion? BrainyBabe (talk) 13:53, 27 September 2008 (UTC)[reply]

Hmm, this ones a bit old (2001) and relates to multitasking, but not the internet etc. From the American Psychological Association: [12]. And heres another paper, this time on gender differences and multitasking: [13] Fribbler (talk) 13:58, 27 September 2008 (UTC)[reply]
I'm still not convinced that Media multitasking is always the same as Human multitasking. Your question assumes so, right? For example, I'm currently doing several things that put me into an "atmosphere of information bombardment" but at this moment, I'm fully focused on typing this post. This is not the same as driving while talking on a phone. Zain Ebrahim (talk) 14:28, 27 September 2008 (UTC)[reply]
I've read the research articles (thanks Fribbler) and they seem to find no significant gender difference. I take the point about different sorts of multitasking; looking after children, in particular, has throughout history been combined with other activities. And boring or silent tasks may well be enhanced by another "information stream" -- like oganised reading aloud to monks and nuns at refectory, or those early communists in quiet factories. But these modern shifts in patterns of attention -- the inability to settle down with a book, for example -- and at another level the very existence of the vulgar dismissive phrase TLDR -- these are subjects under discussion, anecdotally, as I said. I suppose it will all come out in the wash (as long as the Blackbery didn't get inadvetently left in the pocket). BrainyBabe (talk) 16:42, 28 September 2008 (UTC)[reply]

an idle speculation about songbirds

"My mother thinks I'm pretty"

Thinking about the raucous squawks of rosellas and peacocks, I suddenly wondered if there's a general rule there: Do beautiful birds have ugly calls and vice versa?

Adambrowne666 (talk) 12:52, 27 September 2008 (UTC)[reply]

You're probably right in general. If the girl is attracted by one thing why invest much trouble in anything else? That's all evolution will do? Dmcq (talk) 13:05, 27 September 2008 (UTC)[reply]
Presumably all birds find some members of their own species attractive. As for what humans find beautiful, most people don't find crows particularly handsome (although I do), and most people would not describe the crow's call as beautiful. I don't know what sound a Marabou stork makes, but I find it hard to imagine it twittering mellifluously. --Sean 15:26, 27 September 2008 (UTC)[reply]
As a general rule (AFAIK) - small birds sing, larger birds squawk. --Kurt Shaped Box (talk) 16:44, 27 September 2008 (UTC)[reply]
Small birds tend to have to hide. Big birds can strut around and be butch, they don't have to sing to get a mate. As to the maribou stork mom thinking its chick is pretty, whilst we're being anthropomorphic perhaps it thinks that's my boy, no nancies in this nest, you'll beat up the neighours in any fight. Dmcq (talk) 23:31, 27 September 2008 (UTC)[reply]
Different birds cue into different things in song depending on the species. Lady prions are able to assess the quality of a male through the call. In smaller songbirds, which have more complex syrnixes, it is song complexity a dn melody that females like, which is why we like them too, but in other species, like bitterns or Kakapos, it is volume or depth. The srceaches of parrots are not mating calls but contact calls to keep in touch with others of the species by the way, and the contact calls (as opposed to mating calls) of songbirds are simpler and less beautiful even in great singers. Sabine's Sunbird talk 22:17, 28 September 2008 (UTC)[reply]
Fair point, SS - but even the mating calls of parrots are (in every species I've ever heard - aside from the Budgerigar, which is melodic in a twittery way) still loud, harsh and screechy/squawky, albeit less so than the ear-splitting contact calls. --Kurt Shaped Box (talk) 22:37, 28 September 2008 (UTC)[reply]

Thanks, all; I should have guessed it wouldn't be so simple. I'm with you, Toto, I find crows handsome too - their feathers make me think of a phrase used by Lord Dunsany to describe one of his pirates: 'bright black'. Adambrowne666 (talk) 02:27, 30 September 2008 (UTC)[reply]

Uranium nitrate in medicine?

My father went to a homeopath and he prescribed my father a liquid medicine(I presume it is a solution) which had the label "uranium nitrate" on it. Could it be this? Is this compound used in medicine? How risky could it be?Leif edling (talk) 15:19, 27 September 2008 (UTC)[reply]

From homeopathy:
Common homeopathic preparations are diluted beyond the point where there is any likelihood that molecules from the original solution are present in the final product; the claim that these treatments still have any pharmacological effect is thus scientifically implausible
The stuff is probably just water, but on the other hand, why drink water from a quack with a uranium supply? --Sean 15:29, 27 September 2008 (UTC)[reply]

You might not have realized this but homeopathy is a popular method of treatment here in India. So is homeopathy totally pseudoscience?Leif edling (talk) 17:21, 27 September 2008 (UTC)[reply]

Yes. The proposed mechanisms are pure bunk. Homeopathy may have some successes via the placebo effect, or simply by providing generally useful suggestions (go for a walk, drink more fluids, prefer natural to processed foods...), but drinking expensive labeled water with no substance in it is no different from drinking plain water with no substance in it. --Stephan Schulz (talk) 17:38, 27 September 2008 (UTC)[reply]
Additionally, I doubt they would be able to treat your father with uranyl nitrate, it's highly toxic, is a hidious onslaught to your kidneys and is likely to be highly regulated, despite the "dilution" that the above quote states. Stick to real, evidence-based medicine, in my opinion. —Cyclonenim (talk · contribs · email) 17:40, 27 September 2008 (UTC)[reply]
If it's homeopathetic then it is statistically highly unlikely to be even one molecule of the "active" ingredient in the treatment...so it's not going to harm you - it's just not going to do you any good either. That's why they can sell the stuff without going through FDA trials, proving efficacy, etc. What you have there is a $100 bottle of water. SteveBaker (talk) 22:03, 27 September 2008 (UTC)[reply]

Perhaps we can put it this way. If it is not diluted to the point of being just water, it is potentially toxic. If it is diluted to the point of being just water, it's just water. --98.217.8.46 (talk) 18:21, 27 September 2008 (UTC)[reply]

I think the point is though that it seems unlikely that any old quack could get ahold of uranyl nitrate. Given this, it seems likely that the quack is a worse quack then your average quack because the quack didn't even follow the rules of quacks Nil Einne (talk) 19:00, 27 September 2008 (UTC)[reply]
The doc test: If it looks like a quack, swims like a quack and ducks like a quack, then it probably isn´t a doc. --Cookatoo.ergo.ZooM (talk) 19:47, 27 September 2008 (UTC)[reply]

Some homeopathic remedies are not completely diluted. But if any of those are actually helpful, it's by pure freakish chance, and not because the homeopaths have any idea what they're doing. Someguy1221 (talk) 22:47, 27 September 2008 (UTC)[reply]

I agree with the general condemnation of homeopathy, but the logic that all poisons are bad is guilty of the same non-scientific over-simplification of which you accuse homeopaths. My mom started her first round of chemotherapy on Friday. Plasticup T/C 05:12, 28 September 2008 (UTC)[reply]
Well, I'm sorry to hear about your mother. I hope she does well. But I don't see where anyone here was arguing at all that all poisons lack medical usage. There is a big difference between saying that homeopathic dilutions are worthless and saying that all "poisons" are medically useless. --98.217.8.46 (talk) 05:27, 28 September 2008 (UTC)[reply]
While you and your mum have my sympathies and I hope it goes well for her, I would say chemotherapy does actually exemplify the point. There's no doubt all chemotherapies are bad for the human body. The only reason we use them is because cancer is often worse and if used effectively, chemotherapy can help to kill the cancerous cells without killing the person who has cancer. In other words, you need a pretty extreme case where administring such a highly toxic compound as uraynl nitrate is worth it unless the uranyl is so dilute to be mostly worthless. Nil Einne (talk) 17:23, 28 September 2008 (UTC)[reply]

hydrogen storage

im trying to find out if i can store hydrogen with low psi for intake injection on my scooter? because i can produce it in house easier. the model h2 toy rocket kit holds at 3 psi before ingnition. would a small compress still heat it to much? could a modifacation be made to the toy rocket generator? —Preceding unsigned comment added by Fearlss69 (talkcontribs) 18:03, 27 September 2008 (UTC)[reply]

If you have a gasoline-powered scooter, it is not advised that you use other fuels, such as hydrogen gas, to operate it. In the first case, the engine will not run very efficiently, if at all. Secondly, gases, like hydrogen, make poor fuels for internal combustion engines. Hydrogen power is FAR more effective and useful in chemical/electrical engines powered by fuel cells. Hydrogen gas works fine for a single ignition rocket engine, which is basically a single controlled burn. For a gasoline-powered internal combustion engine, which depends on carefully timed repeated explosions, changing to a different fuel will mess with that timing, again, if it works at all. If you are asking if it would be a good idea to strap a hydrogen-powered rocket to your scooter, well, via con dios and have a friend with a cell phone who can dial emergency services when you need it. --Jayron32.talk.contribs 20:22, 27 September 2008 (UTC)[reply]
Also, you might just fry your engine due to hydrogen embrittlement. --Stephan Schulz (talk) 20:32, 27 September 2008 (UTC)[reply]
The problem is that if you store raw hydrogen gas at low pressure, you won't have enough of the stuff to go very far. You need to compress the heck out of it to get enough into your tank to be worthwhile. The answer for storing hydrogen is to store it in the form of metal hydrides and react it in a fuel cell to make electricity. Internal combustion simply isn't a smart way to deal with the difficulties of hydrogen. There are other gasses (propane is certainly one) that internal combustion engines handle rather well - but you still need a high pressure feed. SteveBaker (talk) 21:57, 27 September 2008 (UTC)[reply]
The problem with propane is that our atmosphere doesn't seem to like it. Plasticup T/C 05:14, 28 September 2008 (UTC)[reply]
Yep - indeed. But the amount of atmosphere you can make happy by converting an already rather efficient mode of transport to run on hydrogen is pretty limited...and the effort involved with production of said hydrogen in the world as it is today is pretty much guaranteed to cause greater amounts of atmospheric unhappiness than leaving well alone! SteveBaker (talk) 02:42, 29 September 2008 (UTC)[reply]

Recipe for Life

Hydrogen + Ammonia + Methane + Water + Source of Energy + 4 Billion Years = Me!

Is this more or less correct? Sappysap (talk) 18:35, 27 September 2008 (UTC)[reply]

Well you left out a few molecules. See Abiogenesis#Current_models. --98.217.8.46 (talk) 19:53, 27 September 2008 (UTC)[reply]
You should probably include free oxygen - and stipulate 'liquid' water. There are also a bunch of things you should probably specifically exclude (high acidity environment), etc. SteveBaker (talk) 21:52, 27 September 2008 (UTC)[reply]
Electronic Arts + Spore (game) + a few hours = Me! Plasticup T/C 05:16, 28 September 2008 (UTC)[reply]
I don't think this is very correct. Me is more than just base chemicals - you need billions of years of selectional pressures and environmental factors to get humans. Otherwise, well, you might as well say neutrons + protons + electrons + billions of years = Me.--Fangz (talk) 12:51, 28 September 2008 (UTC)[reply]
23/3 (I should probably offer a prize to the first person to correctly explain this answer) SteveBaker (talk) 02:39, 29 September 2008 (UTC)[reply]
You owe me a beer. DMacks (talk) 04:54, 29 September 2008 (UTC)[reply]
Oooh! You're good! I do indeed owe you a beer...I thought I was going to be safe - but no. Anyway - "Conway's Game of Life" is a computer "game" (well, "Mathematical Recreation" might be a better term) in which things that seem somewhat like life are simulated in the computer. There are actually a whole family of games that use different variations on the rules that Conway wrote. Some of these variations are boring - others are more interesting than the original game. The rules for these myriad variations are encapsulated in just a pair of numbers which is the "recipe" for the game. The recipe for life is therefore 23/3. SteveBaker (talk) 19:18, 29 September 2008 (UTC)[reply]

Plant species

Could someone help me identify this small plant species? Grows on sandy dunes by the sea. Picture was taken in the summer, in southern Portugal. Very common there. Thanks. Húsönd 21:01, 27 September 2008 (UTC)[reply]

Well, it looks like a succulent, so I will guess it belongs to the Crassulaceae. Although without flowers, I can't really tell for sure what family it belongs to. It grows more like a Sedum than anything else, so it might be Sedum sediforme [14].--Eriastrum (talk) 17:25, 28 September 2008 (UTC)[reply]
Hmm, nope, definitely not the species on the flickr photo. The plants growing on the background of the picture I provided are the same as the one being closed up. It is clear that they're not the same kind as the one appearing on flickr. Yet, they do appear to be Sedum. Húsönd 18:41, 28 September 2008 (UTC)[reply]
Does sea spurge, Euphorbia paralias, seem a possibility (photos here and elsewhere on the Web if you Google for "sea spurge")? Deor (talk) 16:06, 29 September 2008 (UTC)[reply]
The more I poke around, the more convinced I become that E. paralias is what it is. Compare your photo with this one, for instance. The plant is native to the Mediterranean region, though it seems to have been spread widely to other areas, where it's often regarded, I gather, as an invasive pest. Deor (talk) 22:20, 29 September 2008 (UTC)[reply]

September 28

slow step of a reaction

why do we use the slow step of a reaction to determine the rate law of it? —Preceding unsigned comment added by 88.242.107.205 (talk) 00:06, 28 September 2008 (UTC)[reply]

Because it's the bottleneck. See Rate-determining step, specifically the funnel analogy. --Bennybp (talk) 00:34, 28 September 2008 (UTC)[reply]
Try this analogy on for size: Lets say you went to a party with your friends across town. Your buddy lives like 4 houses down from you, and you drive over to his house, leave your car at his house, and drive like 5 miles across town to the party. Now, he ditches you, leaving you to walk home. So, you have to be at work the next day, and need to know when to leave the party to get home, shower, and get to work on time. You have 2 steps to consider: The time to walk 5 miles home, and the time to drive your car the 200 yards or so back to your house. Do you even consider how long the 200 yard drive is going to take? No, because its miniscule compared to the time it will take to walk your half-drunk ass the five miles back to your neighborhood (not to mention the time you will be kicking your friends ass the next time you see him). The point is, in a multi-step process, where each step occurs at a vastly different rate, the only thing you really need to care about is the slowest step. All other steps happen so fast that they don't add appreciably to the overall time for the process. In chemical kinetics, the rate determining step is often multiple orders of magnitude slower (in other words, it can take 1000 times longer or more than any other step), so we only care about the slowest step. The other steps will not effect the overall time of the entire process to within the limits of our measuring devices. So we can safely ignore them. --Jayron32.talk.contribs 01:39, 28 September 2008 (UTC)[reply]

Faster than Light Travel

I was recently in an argument with a friend of mine. I remembered reading somewhere (perhaps in my physics textbook) that if you travel at the speed of light, you stop aging. Travel faster than light and you get younger! Is this information correct? And if so what is the scientific explanation for this? —Preceding unsigned comment added by 203.81.220.111 (talk) 06:28, 28 September 2008 (UTC)[reply]

You should read "A brief history of time" by Stephen Hawking. An oldie but a goodie that addresses a lot of stuff like this. Very enjoyable read. NByz (talk) 08:22, 28 September 2008 (UTC)[reply]


SpacetimeNByz (talk) 08:24, 28 September 2008 (UTC)[reply]
Well first, you can't actually reach the speed of light. See time dilation and the twin paradox for the article and the famous example of phenomenon, respectively. It's not as nifty as it may seem; the person who is aging slower is almost frozen in time as far as the person aging faster can see. So unless you just reeaaallly wanna see the year 3000 AD, it's not terribly useful. Maybe it would work as a very inefficient way of keeping your canned peas good for millenia instead of just years, or a deluxe form of cryopreservation without the cold part... Someguy1221 (talk) 09:23, 28 September 2008 (UTC)[reply]
No, it's not correct. There's a certain restricted sense in which "as you approach the speed of light time goes slower." In the limit of v = c "time stops" (though you can't reach that limit). Some people imagine that when v > c time would go backwards, but that's wrong: the formula for the amount of the slowdown is , which doesn't go negative for v > c. Another thing people sometimes fail to appreciate is that the slow time applies to everything; you age "slower" but also think "slower," so you experience a normal lifespan, not a lengthened one. I put "slower" in quotes because, given that everything slows down equally, it's hard to say what it's slower than. -- BenRG (talk) 10:40, 28 September 2008 (UTC)[reply]
Though if you do have something that is v > c, I seem to recall its worldline does go backwards from certain reference frames, or something like that (like a tachyon). (But I agree this isn't the same thing as saying something is really traveling back in time at all, much less "getting younger".) --98.217.8.46 (talk) 15:11, 28 September 2008 (UTC)[reply]
"Tachyon" is merely a name for some hypothetical thing - they don't exist outside of Star Trek and the math says they can't exist - so let's stop worrying about them. The equation produces the square root of a negative number if v>c -- there are no calculations in the "real world" where the result is a complex number. Even if there were - it's not at all clear what multiplying your age by a complex number would actually mean - but "getting younger" is certainly not it. Essentially, the math fell apart - and that only happens when something impossible is going on. It's a pointless speculation - you can't go as fast as the speed of light if you have any rest mass...so no...just no. SteveBaker (talk) 02:32, 29 September 2008 (UTC)[reply]
Another very dumbed-down way I've heard this explained to non-science guys (so, you science guys can skip this so you don't get worked up by it)... By speed, in this case, you are referring to a ratio between your energy and your mass. To speed up, you can add energy, remove mass, or convert mass to energy. The only way to reach the speed of light is to be 100% energy. Since adding energy will not get rid the mass, the only way to do this is to either remove all mass, leaving only the energy, or to convert all the mass to energy. We do both very well. When heat is emitted by your body, it basically separates itself from your mass and becomes 100% energy. You (the mass) don't get any special benefit from it. Also, if you stand in the middle of a large thermonuclear explosion, your mass will quickly and efficiently be converted to energy. You'll suddenly be travelling at the speed of light. However, you won't be you anymore since there is no means of collecting that energy and putting it back together again to recreate you. So, again, you don't benefit much from the experience. -- kainaw 02:41, 29 September 2008 (UTC)[reply]
No explosion of any kind (even a supernova) is able to convert a person (or anything else macroscopic) entirely into energy; see baryon number. --Tardis (talk) 15:41, 29 September 2008 (UTC)[reply]

Protons and Electrons

Are there equal numbers of protons and electrons in the universe? --220.237.34.141 (talk) 07:32, 28 September 2008 (UTC)[reply]

No. An electron and anti-electron can combine to produce a photon so that's one less of them for a moment for instance. There is charge conservation though. Dmcq (talk) 10:37, 28 September 2008 (UTC)[reply]
Why would the reaction of an electron and an anti-electron be faster (or occur more) than proton + anti-proton?--220.237.34.141 (talk) 11:30, 28 September 2008 (UTC)[reply]
Protons and electrons are really very different things. There's little reason to think that many things will happen the same for both. I suspect there's more electrons than protons, actually - electrons are produced naturally through, for example, pair production. Protons are generally not produced in the same way, because protons are much more massive than electrons and hence require a lot more energy to produce. And lets not get started on virtual particles.--Fangz (talk) 12:47, 28 September 2008 (UTC)[reply]
I wasn't saying such a reaction occurred more often. Simply that if the numbers were equal and this reaction happened then afterwards they would be unequal. Another reaction might make them equal afterwards but overall in the universe even if the numbets are roughly equal then the odd they are actually exactly equal for any length of time is very very very small indeed. Or perhaps you were thinking almost equal rather than equal? You'd then have to specify 'almost' but I suspect the answer even then as Fangz says is no. Dmcq (talk) 13:16, 28 September 2008 (UTC)[reply]
I strongly disagree. If the numbers of protons and electrons were significantly unequal (suppose there were more electrons) - then wouldn't any significant imbalance result in there being a huge negative charge on every macro-scale object? This charge would cause objects to repel each other...and because electromagnetic forces are VASTLY stronger than gravity - wouldn't this cause the immediate spontaneous disintegration of any large objects? The only way I could imagine this NOT happening would be if all of space were uniformly packed with electrons so that all of the repulsions would balance out. But that clearly isn't the case - so why isn't the sun repelling the earth with an ungodly force? No - I think the numbers of protons (or other positively charged particles) must almost exactly equal the number of electrons (or other negatively charged particles)...I don't see how the universe could function otherwise. SteveBaker (talk) 02:24, 29 September 2008 (UTC)[reply]
Sorry yes, thinking about it you're probably right. The main place charged particles besides electrons and protons would occur is inside stars but the sun for instance doesn't actually have much happening in it - the reactions like photon to electron and positron or muon conversions are rare compared to the total numbers of ionized atoms and electrons. So the numbers would be roughly equal. Dmcq (talk) 13:48, 29 September 2008 (UTC)[reply]

Einsteins starting point

Where did Einstein start when he formulated his special theory of relativity? For instance, e=mc^2, where did he derive this equation from? Related question, what evidence did he have, if any, that his formulae were correct? How did he know that the direction he was going in was the right one? Cheers 59.100.200.162 (talk) 14:09, 28 September 2008 (UTC)[reply]

This is a big question. You are probably better off getting an actual book about this than asking us to muddle you through the history of it, which is detailed and complicated. We have a longer article on this (e.g. History of special relativity) but in glancing over it right now, it seems fairly unreadable unless you already know all of the physics involved (and is in my opinion historically dubious—it is history the way a scientist tells it, not how a historian tells it, and makes it look like the history is just a bunch of equations being thrown around in a vacuum).
Conceptually the starting point is just the question of how you can reconcile two things Einstein and his contemporaries knew to be true: the speed of light is independent of the speed of its emitter (it is constant in a vacuum), and that Galilean relativity should hold in all physical frame of reference (Galilean relativity is just why you don't feel like you are moving when you are sealed inside the belly of an aircraft, for example, and why you don't feel like you are moving when you are standing on a rapidly rotating planet—in a wholly-enclosed constant frame of reference you cannot know your own absolute speed by simply experiential means). Both of these facts were things that his contemporaries would have found fairly obvious, but wouldn't have necessarily put them together at the same time.
It is in the reconciling of those two facts that most of SR comes out of—if the speed of light is really independent of the speed of its emitter, despite the fact that physical frames of reference will be relative, then two observers traveling at different speeds will measure the speed of light to be the same no matter what their different relative speeds are, and if that is the case, then all sorts of interesting implications come up about time, space, and etc. I won't go through all of those implications here but that's really where it becomes clear that space and time are deeply, deeply linked.
This is a very poor gloss on what Einstein was really doing and really thinking, though. Again, a good historical book (Walter Isaacson's recent biography is fairly good on this point) will illuminate much more clearly this sort of thing. There were a wide variety of factors relating to Einstein's previous education, his work at the patent office in Bern, his meetings with creative friends, the books he read, and so forth.
As for evidence—for much of Einstein's work, there was not any immediate evidence other than the fact that the work was actually conceptually quite simple and the reasoning seemed to work out. But by itself nobody was all that convinced that it was too important or anything other than playing with math. Most of SR was not proven by experimental means until very late in the 20th century, where observations of things like muons made it clear that indeed going very fast can affect size and time. Einstein did not think all of this work was revolutionary—it just made sense. His work on the quanta was much more revolutionary, and garnered much more attention from other scientists. Only after creating General Relativity, and having a few surprising experimental confirmations, did Einstein become known as a very important figure in physics. --98.217.8.46 (talk) 15:06, 28 September 2008 (UTC)[reply]


E=mc² didn't appear in the original paper on relativity, it's from a later paper (though published the same year). There was ample experimental evidence for Einstein's ideas at the time of publication, and in fact all of the math of the theory had already been worked out by Lorentz and Poincaré and others. Einstein's contribution wasn't the math, it was a new philosophical framework for measurement—all the stuff about clocks and metersticks. Prior to 1905 everyone had assumed (without really thinking about it) that there was nothing problematic in principle about synchronizing clocks, though it might be tricky with real, imprecise instruments. Einstein was the first to realize that (a) this was not a philosophically necessary position and (b) it wasn't actually true. I think this was obvious to most physicists once it was pointed out, and the typical reaction to the paper was "of course, how could we be so stupid" and not "that's a strange idea, I wonder if it's true." -- BenRG (talk) 15:12, 28 September 2008 (UTC)[reply]
Well and most of the physicists at the time thought Einstein was just being clever with his reasoning—not that it was really very fundamental work. The recognition that the 1905 papers were something amazing didn't come until later. (Quanta notwithstanding—that was taken up by Planck and others.) --98.217.8.46 (talk) 15:15, 28 September 2008 (UTC)[reply]
Gary Larson suggested that it was actually a custodian who was key to getting the equation we now know. Can't find a copy of the diagram explaining it right now... DMacks (talk) 20:28, 28 September 2008 (UTC)[reply]
I found [15]. PrimeHunter (talk) 02:40, 29 September 2008 (UTC)[reply]
Take a look at how close Friedrich Hasenöhrl came in 1904. He had everything but the coefficient. --Arcadian (talk) 00:16, 30 September 2008 (UTC)[reply]

Nightmares

I heard once that being overheated can contribute to the likelihood of nightmares, and it seems to be true for me. Does anyone have information on that? --Masamage 15:31, 28 September 2008 (UTC)[reply]

I've no idea, but I do want to point out that for me, anecdotally, I've noticed that placebo effect on nightmares seems rather significant. I had the idea when I was a kid that if I crossed my arms like a corpse I would have nightmares (hey, I was very young!), and sure enough, as long as I believed it made sense, it worked. When I got old enough to realize how daffy that was, it stopped working. --98.217.8.46 (talk) 15:39, 28 September 2008 (UTC)[reply]
More anecdotes, but overheating -> nightmares does fit a pattern I've noticed. It seems to me that my most vivid dreams come when I'm not deeply asleep; that is, when I'm drifting off or just waking up. Perhaps hypnagogic dreams are an influence there. A more widespread notion is that eating certain foods (like cheese or spicy stuff) can increase bad dreams and that could also contribute to broken sleep. Matt Deres (talk) 16:24, 28 September 2008 (UTC)[reply]
I would say this is one area where it's rather hard to rule out other factors, like the fact that you are 'overheated' means you're probably not going to sleep comfortably, which probably means you're going more likely to remember any dreams, nightmare or otherwise Nil Einne (talk) 17:15, 28 September 2008 (UTC)[reply]
Very true. What may be causing any "overheating" may also be causing nightmares. --Russoc4 (talk) 18:07, 28 September 2008 (UTC)[reply]

Interesting. So, can I take this to mean that physical discomfort in general can increase the chance of having dreams with unpleasant elements? (In the case that's making me ask about it, I think I would have remembered it regardless, because the bad thing in the dream caused me to physically struggle and that's why I woke up.) --Masamage 01:40, 29 September 2008 (UTC)[reply]

Generally dreams occur in the lighter phases of sleep, the so-called REM phases. If your sleep is disturbed, by heat, cold or indigestion, for example, then you are less likely to achieve deep (non-REM) sleep and thus experience more dreams. It is possible that being hot may influence the subject matter of your dream but I think the theme of a dream is more closely linked to your present psychological state. 86.4.187.55 (talk) 10:02, 29 September 2008 (UTC)[reply]

Tropical Cyclones sorces

What are other good sources for hurricanes besides the National Hurricane Center's Tropical Cyclone Report.--Leave Message orYellow Evan home 16:11, 28 September 2008 (UTC)[reply]

For Atlantic and East Pacific storms, the TCR is the cat's pajamas. But what sort of information are you looking for? Meteorological data? Damage assessments? Predictions? Social attitudes? Government responses? International aid? Demographic data? Seasonal summaries? Historical trends? Economic impact? Cultural impacts? Impact on immigration? You'll need different sources for all of these things. Plasticup T/C 18:01, 28 September 2008 (UTC)[reply]
A separate but related question: "Cat's pajamas"? That's a new one to me. But I kinda like it.-RunningOnBrains 23:33, 28 September 2008 (UTC)[reply]
The question is whether the cat's pajamas is better than the bee's knees... --Jayron32.talk.contribs 02:25, 29 September 2008 (UTC)[reply]
They are both the dogs' bol...oh - wait, there are Brit's here who would understand that one! SteveBaker (talk) 02:53, 30 September 2008 (UTC)[reply]

Let's suppose I have a pencil (or a pen, or a wristwatch, or any small object) and I want to completely disintegrate it, destroying every one of its atoms. Can it (theoretically) be done? If so, how? Is it possible to destroy every atom? Thanks in advance. —Preceding unsigned comment added by XxCutexXxGirlxX (talkcontribs) 17:01, 28 September 2008 (UTC)[reply]

You could get an antimatter pen Nil Einne (talk) 17:14, 28 September 2008 (UTC)[reply]
You could throw it into the sun and let nuclear fusion take its course.
Or you could throw it into a blackhole and allow it to be crushed into a singularity and then evaporated as Hawking radiation. The mass would still exist, but the original atoms would be destroyed forever. APL (talk) 18:55, 28 September 2008 (UTC)[reply]
you could just sit back and wait a few months for the black hole at the LHC to suck it in. 96.231.83.176 (talk) 20:02, 28 September 2008 (UTC)[reply]
  • Throwing it into the Sun won't work. The Sun is only hot enough to fuse hydrogen into helium, not hot enough to destroy all sorts of atoms. (Well, their electrons would be stripped off, but I assume you want the nuclei destroyed.) You need the sort of temperatures found only in a supernova. --Anonymous, 01:35 UTC, September 29, 2008.
A neutron star would be just the thing. None of that hassle with Hawking radiation shooting back the energy of your ex-girlfriend's photo right back at you...forever reminding you...taunting, taunting... SteveBaker (talk) 01:42, 29 September 2008 (UTC)[reply]
You could wait. In a mere 1040 years, all the protons in the pen should have decayed into mostly neutral pions and positrons. Of course, proton decay is only theoretical. If it doesn't exist, you'd have to wait a bit longer (still only 101500 years), at which point all the atoms in the pen should have become iron-56.-RunningOnBrains 23:31, 28 September 2008 (UTC)[reply]
In old science fiction films, the spacemen carried disintegrator guns. Perhaps one of these gadgets could do the job. Edison (talk) 00:17, 29 September 2008 (UTC)[reply]
More seriously, it is theoretically possible (an practically it has been done for some types of atoms) to split large atoms (like metals and carbon) into smaller atoms. If you can do it for one atom, you can do it for each and everyone of them. But you would still have some residual mass of matter. If you want to transform the whole thing into energy, and be left with no mass, then your best bet is antimatter. But why would you want to do that It would require a crazy amount of energy, instead just cuting the link between each of these atoms would make your object puff into various gases without the complexity of spliting the atom --Lgriot (talk) 01:27, 29 September 2008 (UTC)[reply]
Most of the above are impractical and could not be done on earth. Your best chance may be to insert into a atomic accelerator synchrotron or Large hadrion collider and zap it with fast moving nuclei, to break up the nuclei. It will be tough to destroy the protons in the hydrogen, so probably immersion in a nuclear reactor could saturate it with neutrons and covert it to deuterium. Graeme Bartlett (talk) 02:33, 29 September 2008 (UTC)[reply]
Send it to a priest and have him do some transubstantiation on it. It might look the same but it will really be different. Dmcq (talk)
Um, the annihilation caused by an pen/antimatter pen meeting would wipe out probably the entire continent of North America and points south (depending on where you chose to do this). Just FYI that that's really not a viable way to disintegrate the pen. The black hole thing would be easier.  :) 31306D696E6E69636B6D (talk) 13:23, 29 September 2008 (UTC)[reply]
No it would only be equivalent to about the explosion of a small hydrogen bomb. See Mass–energy equivalence, the Nagasaki bomb would have converted about a gram of mass to energy, and when I weighed my biro I could barely make out any movement on the kitchen scales. Not tha I want you doig the experiment anywhere near me NIMBY Dmcq (talk) 14:09, 29 September 2008 (UTC)[reply]

Uranium in homeopathic medicine

I asked a related question yesterday and now I've found this: [16]. It's weird; what's with the "make uranium at home" and "homeopathic uranium" statements? Is this the limit of how dangerous homeopathy can get? Leif edling (talk) 17:15, 28 September 2008 (UTC)[reply]

Actually, this sounds almost completely harmless. Crushed "red rock" will bleed negligible amounts of uranium (if any) into the water, and will be quite unlikely to be otherwise harmful. It won't do anything positive, either, but the greatest risk is probably hitting yourself on the thumb while crushing the rock. --Stephan Schulz (talk) 18:01, 28 September 2008 (UTC)[reply]
Furthermore, if the homeopathic "remedy" is prepared "properly", the end user is unlikely to be in any danger; thanks to Avogadro's number, all he's getting is water. (Or whatever they're using as the diluent.) -- Captain Disdain (talk) 01:32, 29 September 2008 (UTC)[reply]
It's really impossible to make sensible statements about something as obviously stupid as homeopathy - but proponents never properly answer any of the most obvious concerns. A sure-fire test for a ridiculous theory is that it typically falls apart even worse when you believe what the nut jobsexperts in the field are claiming. So let's do that...
Let's suspend disbelief for a moment and assume that homeopathy really worked. I mean, really: We're told that the uranium (in this case) left an "imprint" in the water that did something theraputic - and that this effect gets stronger the more you dilute it. My first concern is how come the few silicon atoms that washed off the glass you mixed it in didn't do something? What about the two human skin cells and the million or so bacteria and viruses that inevitably got in there during the process? Water is NEVER 100% pure - so isn't it going to be "imprinted" with countless other things that it came into contact with? The purer the water you start with - the stronger the effect of the impurities are claimed to be - so using super-clean distilled water only makes the 'pollution' in it even more powerful.
This stuff (if you carefully examine the bottles on the shelves at WalMart) is claimed to have a shelf life of at least a year or two - so the "imprinting" in the water molecules has to last at least that long. Where do they get their "non-imprinted" water? There are claims for homeopathic "cures" for low blood pressure and quite different cures for high blood pressure...but how do you make sure that the imprinting for the wrong "cure" wasn't in the water to start with? You really have no clue whether your water came into contact with a (whatever cures low blood pressure) molecule a couple of weeks before you decided to make a high blood pressure cure with it.
What's worse, they claim that the more dilute you make the stuff, the stronger it becomes - but the prior imprinting imposed on the water BEFORE you start your titration is guaranteed to be made yet stronger by your titrations. Since they dilute (say) uranium to the point where statistics and Avagadro's number proves that there is none of the original stuff left, absolutely ANYTHING that was in the water at least as long as a year ago will be having a super-powerful medicinal effect - even if the water is utterly, 100% pure! I imagine that they get their original distilled water from normal sources - so a year ago, your water was probably a part of the ocean - or floating around in a cloud someplace.
In this particular case, you're being asked to take some reddish granite and use that as the starting point "because it contains uranium" - but it's "rock" - it contains hundreds of other kinds of atoms and molecules. If homeopathy really works - then ALL of those things are also being used to "imprint" the water with whatever things they do to you. The "researchers" in this field cannot possibly have tested all of those other molecules in proper human trials - how could they possibly know that your local flavor of granite isn't going to imprint something really nasty into the water?
The claim is always that homeopathic medicines are 100% safe (which they certainly are!) because - for some inadequately explained reason - they only have "beneficial" effects. But how does the underlying chemistry, physics and biology "know" what is "good" and what is "bad". To cure you of a bacterial infection - it has to kill bacteria. How do these homeopathic imprintings "know" to cure the human and not to cure the bacterium?
The whole theory falls apart so seriously under the most gentle nudges from a scientific mind that answering questions about homeopathy is truly impossible to do - other than to state the bloody obvious - which is "IT CANNOT POSSIBLY WORK - EXCEPT BY THE PLACEBO EFFECT". If it does work by the placebo mechanism then why bother with all of the titrations and other bullshit. Just start a new "cult" that says that drinking water from a magic spring will cure you of everything...oh - but wait - the religious nut-jobsexperts already came up with that one.
SteveBaker (talk) 02:13, 29 September 2008 (UTC)[reply]

Answering questions about homeopathy is no different than answering questions about what effecy the witchdoctor's dance is going to have. The basic question is so patently rediculous to anyone that even paid attention in high school that when you say "your just drinking water", people say "no, that can't be it!" and try to find more. Let me restate it. You are just drinking water. If you have a genuine medical concern, find someone who has undergone training in the operation of the human body. If that doesn't work, then ooo eee ooo aaa aaa, zing zang walla walla bing bang... --Jayron32.talk.contribs 02:19, 29 September 2008 (UTC)[reply]

Wow! My splitting headache just went away just by reading that! I'm going to cut and paste it and sell it for $100 and there is nothing you can do to stop me because it's under GFDL! :-) SteveBaker (talk) 03:00, 29 September 2008 (UTC) [reply]
There's nothing particularly mysterious about homeopathy, the placebo effect is well-documented. It seems as though when someone you trust carefully investigates your symptoms and says "take this, it will cure you", in (I think) about 30% of cases above the baseline, the symptoms are alleviated to at least some degree. The homeopathic concept itself is full of crap, but the placebo effect is a mystery that we should be investigating further. Much as I detest the junk spouted about succussion, there is something to the concept of having individual attention paid to one's health. That of course is what homeopaths do, they draw up an individual profile - which of course renders meaningless the prospect of controlled random-double-blind studies - but also renders inaccessible further studies of the placebo effect. Franamax (talk) 03:12, 29 September 2008 (UTC)[reply]
While it may be true that the placebo effect has 30% over baseline positive results, (and I am not sure its that high, but I will accept it axiomatically for the sake of the arguement) any intentional treatment is bound to have much better results. This is especially if your measure for success is that "symptoms are alleviated to at least some degree", since any intentional treatment will already have the placebo effect built in (as long as the patient believes in the cure, the placebo effect should be additive to the theraputic effect of any treatment). While the placebo effect is real and dramatic, its theraputic value is negligible since its effect is true for both real and imagined cures, and the real cures have the added benefit of actually having mechanisms shown to affect the complaint in question... --Jayron32.talk.contribs 03:22, 29 September 2008 (UTC)[reply]
Don't forget that part of the placebo effect is measurement error. (That's why you see a placebo effect even in veterinary trials even though the cats and dogs involved certainly don't believe that a trip to the vet will cure what ails them.) I would hate to undergo a treatment whose main advantage is that it makes my doctor think I've improved. APL (talk) 13:47, 29 September 2008 (UTC)[reply]
The control group will have the same measurement error so that won't affect the conclusions. --Tango (talk) 15:07, 29 September 2008 (UTC)[reply]
Of course. All placebo effect occurs in both control and experimental groups in a properly done experiment. Jayron32 was talking about the "theraputic value" of placebos, I was pointing out that even if placebo effect seems to cause a 30% recovery, only a portion of that will be actual recovery, the remainder will be measurement error. APL (talk) 16:49, 29 September 2008 (UTC)[reply]
No, if your experiment is intended to measure the therapeutic value of a placebo then only the experimental group would get the placebo, the control group would get nothing (it is important to never use the results of an experiment for any purpose other than that for which it was intended - trying to measure placebo effectiveness from the results of an experiment comparing a placebo with a new drug is never going to work). The measurements would, however, be the same for both groups, so the error would not effect the conclusion. Also, you're assuming the error would be towards greater affect rather than lesser, what is your reasoning for that? --Tango (talk) 22:27, 29 September 2008 (UTC)[reply]
Even that is a slightly tricky experiment. We no longer do "blind" studies (in which the patient doesn't know what he's getting - but the doctor does) - we do "double-blind" studies where the doctor, the patient - and even the experimenter doesn't know who gets what until the study is over. For a placebo to work, it may be necessary for the doctor administering it to be unaware that it's a placebo in order that he can exude confidence that it's actually going to work. That's a tough call in the "nothing vs placebo" case because the doctor knows that it's not a real drug-versus-placebo study because half of his patients are getting nothing whatever. To do it right, you need THREE groups - one gets the placebo, another gets a real drug and the third gets nothing. The guys who are getting the real drug are needed in order that the doctor may believe that he's really treating his patient when he's actually handing out the placebo.
The problem with homeopathy is that it's not just a way to give a patient a placebo - they really do go to all of the ridiculous trouble to dilute the 'active ingredient' a bazillion times - and that costs real money. If you're going to use an expensive process to manufacture your placebo then you might as well use a real drug and get the benefit of placebo PLUS a real drug.
The homeopathic thing that annoys me beyond rage is homeopathic pet treatments. Your pet isn't getting the benefit of the placebo effect - all that's happening is that the owner is spending a lot of money and completely FAILING to treat the animal. That's just horrible. SteveBaker (talk) 23:30, 29 September 2008 (UTC)[reply]
You're right, you do need a 3rd group if it's going to be double-blind (which it needs to be). The results from the 3rd group wouldn't be used in forming a conclusion about the effectiveness of placebos though, which makes for a very strange study! Such a study can get other results as well, though - it can show what proportion of the improvement seen in the people taking the drugs was from the drug, what was from the placebo effect and what was just natural improvement. Apparently a study done on antidepressants showed that most of the improvement was natural, most of what was left was placebo and just about 10% was the drug (still statistically significant, but very small). --Tango (talk) 23:36, 29 September 2008 (UTC)[reply]
Hmmm - perhaps a yet better study would be to take two groups of patients and doctors - hand out identical placebo to both groups but "leak" the news that the drug is really a sugar pill to half of the people. If the ones who know they are on placebo don't do as well - then there is definitely something to it. SteveBaker (talk) 02:51, 30 September 2008 (UTC)[reply]

Hydrogen Bonding

Which would require less energy, moving two particles of the same charge close together, or moving two particles of opposite charges close together ? Why? —Preceding unsigned comment added by 24.215.48.99 (talk) 20:25, 28 September 2008 (UTC)[reply]

Visit our Coulomb's law page. DMacks (talk) 20:56, 28 September 2008 (UTC)[reply]
Your question is a bit vague. I can interpret it in any of 3 ways:
  1. You have 2 particles of identical charge that are X distance apart, and 2 particles of identically opposite charge that are also X distance apart. You try to move them against the natural attraction or repulsion of their charges. At the instant you try to move them, which requires more energy. Well, in the first timeless instant when you move either, the answer is they require the same energy BUT...
  2. In the same situation, the two oppositely charged particles will require less and less energy to move because the distance between them (and thus their attraction for each other) is decreasing, BUT the two same charged particles are getting closer, and thus get harder to push. Thus, for any meaningful time interval (that is for any delta-t greater than 0) the same charged particles require more energy.
  3. If you are asking what will happen if you push the particles towards each other in both instances, then the answer is that the same charged particles will require more energy, while the opposite charged particles require NO input of energy (they move spontaneously)... In fact they will generate excess energy equal in magnitude to the energy you had to put into the same-charged system to make them move the same amount (assuming of course no friction).
Hope that helps. --Jayron32.talk.contribs 03:15, 29 September 2008 (UTC)[reply]

dynamics

suppose you have 2 forces pulling a box, the box is resting on a floor with friction, if the box is not moving because all forces are at equilibrium, then do we still need to worry about friction and include it in the calculation? —Preceding unsigned comment added by 142.151.132.11 (talk) 20:34, 28 September 2008 (UTC)[reply]

Is friction a force on the box? If so, does it affect how the box moves when it is pulled? If so, then it is a force that affects the box's motion in response to pulling, and therefore must be included in calculations of that motion and the forces involved. DMacks (talk) 20:48, 28 September 2008 (UTC)[reply]

the static friction is on the box, but my problem is, in which puller's direction is the frictional force going towards? —Preceding unsigned comment added by 142.151.132.11 (talk) 21:00, 28 September 2008 (UTC)[reply]

Friction opposes "motion", so I'd think consider it to oppose whatever motion would be happening otherwise (i.e., opposite the net force of the pullers). DMacks (talk) 21:38, 28 September 2008 (UTC)[reply]
I don't think it changes the result of the calculation, but it could be more complicated than that: if the box is under tension, the friction force could be in both directions on different parts of the bottom of the box. Its sum will be the same as the single force you can calculate instead. --Tardis (talk) 23:09, 28 September 2008 (UTC)[reply]
Since the OP didn't say otherwise, I think it is probably assumed that the box is rigid and you can ignore things like tension. --Tango (talk) 23:27, 28 September 2008 (UTC)[reply]
I imagined that a more correct answer might be at least interesting (to the OP or someone else) if not necessary. It's not entirely irrelevant: after all, for a stiff box the structural distortions associated with the tension and with the friction are of the same general scale. --Tardis (talk) 23:43, 28 September 2008 (UTC)[reply]
In the real world (as opposed to a physics textbook) it could take a larger force to start an object moving than to keep it moving. Edison (talk) 00:14, 29 September 2008 (UTC)[reply]
What you have to do is to resolve all of the non-frictional forces on the box - you can reduce them to a single 'net' force. Now you have an easy problem - is the net force able to overcome friction? What gets double-ikky is that for most real-world surfaces, the 'static' friction is higher than the 'dynamic' friction - so once you've established that the static friction is overcome, you have to do the math again with the dynamic friction in order to determine the final motion of the box. Yukky! SteveBaker (talk) 01:37, 29 September 2008 (UTC)[reply]
It's also worth keeping in mind that the rule you learn at school — that friction equals the normal force times a coefficient of friction — is only an approximation of a very complex interaction between two surfaces. As it happens, for a lot of practical situations that approximation works remarkably well for its simplicity, at least as long as you remember to use the right coefficient depending on the nature of the contact between the surfaces (static, sliding, rolling, etc.), but one should not mistake it for any kind of "fundamental law". It's a fairly good empirical rule of thumb, nothing more and nothing less. —Ilmari Karonen (talk) 16:32, 29 September 2008 (UTC)[reply]

September 29

Oceanic Mammalian Question

I reckon that whales, seals, and dolphins have larger lungs than humans that allow them to dive for such long periods of time. However, they also have larger bodies than humans! Do they also have more efficient lungs which better oxygenate their bloodstream? Is that even a real thing?

Thanks! —Preceding unsigned comment added by 76.24.15.197 (talk) 02:33, 29 September 2008 (UTC)[reply]

Our article on whales simply states: "Whales have a unique respiratory system that lets them stay underwater for long periods of time without taking in oxygen." The dolphin article doesn't provide any more insight. -- kainaw 02:45, 29 September 2008 (UTC)[reply]
Dolphins aren't much bigger than humans - their lungs are probably no bigger. I suspect (without more knowledge) that their metabolism is designed such as to allow the animal to continue to function with much lower blood-oxygen levels than humans. But are dolphins really much better at this than humans? Tom Sietas can hold his breath underwater for over 15 minutes (although he cheats a bit by pre-breathing pure oxygen first - his "fair" record is a little over 10 minutes). One assumes that most humans could come close to that ability if they trained to do it since birth as dolphins are. Bottlenosed dolphins can only hold their breath for 30 minutes. So the disparity isn't as great as you might think. SteveBaker (talk) 02:57, 29 September 2008 (UTC)[reply]
I could be wrong, but don't most of the records just involve people holding their breath underwater for long periods of time? They don't involve people actually swimming or otherwise doing stuff underwater for that time and I'm not convinced the record will be as long if it did. With cetaceans though I think they maintain mostly the same level of cognitive/mental and physical functioning throughout the time of holding their breath Nil Einne (talk) 08:52, 29 September 2008 (UTC)[reply]
This is probably closest to what we want [17] 244 metres doesn't seem a great distance to me. Since time isn't a concern, it isn't mentioned but if we take double the world record World record progression 200 metres breaststroke that's only about 5 minutes. And even if the time is more then that, it would suggest the person is far from at optimal physical activity level Nil Einne (talk) 09:00, 29 September 2008 (UTC)[reply]
Actually the current record is 248m [18]. Also, I just remembered I forgot that was with fins, so they naturally should be faster then the world record set by someone with pseudofins so 5 minutes is definitely probably quite generous. Nil Einne (talk) 09:10, 29 September 2008 (UTC)[reply]
The discussion seems to be in the Cetacea article instead. It mentions higher myoglobin concentrations in muscle; myoglobin retaining oxygen better than haemoglobin. And argues that in addition, having more muscle mass (ie from larger body size) increases the oxygen storage capability because muscle stores oxygen, and because larger animals have slower metabolic rates.WikiJedits (talk) 12:54, 29 September 2008 (UTC)[reply]
Sperm_Whale#Diving_and_breathing has a bit, too, including some interesting stuff about bone-pitting due to decompression. --Sean 13:57, 29 September 2008 (UTC)[reply]

MATLAB question

I want to take an arbitrary length array (say, for example, like [1 2 3 4 5]) and convert that into an anonymous function with a function.

So I have a function called any_polynomial which will take an argument of such an array and return the anonymous function, which can be assigned a function handle. This is for an assignment, so I only really want a hint or two. The assignment tells us to use polyval, which is quite useless for this in my opinion, as then the anonymous function will be something like "@(x) polyval(input_array,x)". Thoughts? --M1ss1ontomars2k4 (talk) 03:56, 29 September 2008 (UTC)[reply]

I'm not sure if this is an appropriate question for the science reference desk (it being about Matlab and being homework), but I'll try to help a bit. First of all, I wonder if you haven't misinterpreted the assignment - the function polyval(p,x) evaluates a polynomial at x that has the values p(n) as its nth coefficient. I'm not sure what you mean when you say you want to "convert an array into an anonymous function". I guess what I'm saying is...the anonymous function you have described certainly does something, but it's not clear to me why you don't like it. Could you be more specific about what you want the anonymous function to do? --Bmk (talk) 04:58, 29 September 2008 (UTC)[reply]
Well, the science ref desk has "engineering" as one of the subjects that fits under it, and this class is taught by the MechE department. It doesn't have much to do with math or actual computer science, so I thought I'd find the best help here. At any rate, the questions asks us to basically take [1 2 3 4 5] and get @(x) x.^4 + 2.*x.^3 + 3.*x.^2 + 4.*x + 5. That's how I interpreted it; it actually asks for the polynomial to be returned, which I assumed meant the actual polynomial. At any rate, I've largely concluded that it would be really not worth the effort or the last whatever percentage of my grade to come up with such a ridiculous bit of code that has almost no real practical usage under any circumstances. Also, the next part of the assignment asks us to use our new function with fzero to find roots, so I explained in the comments for my code that, while the polynomial is not returned, something just as good is--it's a function that represents the polynomial and can be used with fzero. Thanks anyway! --M1ss1ontomars2k4 (talk) 06:01, 29 September 2008 (UTC)[reply]

Flatbed scanner covers

Why do flatbed scanners have covers? I have tried scanning with the cover open and with the cover closed, and it seems to make little or no difference in the quality of the scans, even when the document does not cover the entire scanning surface. —Lowellian (reply) 08:50, 29 September 2008 (UTC)[reply]

To reduce the light bouncing out and being annoying. If you're sat in a room scanning a document while working on something on your pc the last thing I could imagine wanting is a overly-bright light moving across the room from my scanner. I would have thought it also could make a difference with some documents scanning-quality but if for nothing else I would suspect it is just because the light is quite strong and it's better to try cancel it out with a simple sheet of plastic rather. Oh and it probably acts as a useful guard against damage occuring to the glass plate when the scanner isn't in use. 194.221.133.226 (talk) 08:53, 29 September 2008 (UTC)[reply]
On the contrary, I actually find it more annoying to close the cover since by not bothering with opening-and-reclosing the cover between each document, I can scan considerably faster and with more ease. However, the instruction manual seems to stress over and over again how important it is to close the cover. That's what I don't understand: is it somehow harmful in some way not to close the cover? If it isn't, why does the instruction manual stress closing the cover so much? —Lowellian (reply) 09:04, 29 September 2008 (UTC)[reply]
Aren't they at least partially there to help squash the document if it isn't flat, e.g. a book? Admitedly most dekstop scanner ones are so flimsy as to be almost worthless in that regard although they may allow you to apply more even pressure if you are holding it down. Also, I think there may be some difference if your scanning a thin single page and the scanner light is very bright Nil Einne (talk) 09:15, 29 September 2008 (UTC)[reply]
At least older scanners and copiers would copy any part of the surface not covered by the original or the cover in black, thus wasting a lot of toner. Some of the more modern ones may do some software-based cropping, but manuals change slowly (AFAIK, Apple still sends out battery care instructions that made sense three battery technologies ago). I'd also expect that there is some workplace safety issue with the bright light. It probably does not matter for people who only scan a few pages, but imagine someone doing nothing but scanning day in and day out. --Stephan Schulz (talk) 09:28, 29 September 2008 (UTC)[reply]
That doesn't happen for me. The scanner and software I'm using always properly detects the edge of the paper, with or without the cover.
But wait. "Workplace safety issue"? So is the light dangerous in any way, or is it just ordinary light? —Lowellian (reply) 19:19, 29 September 2008 (UTC)[reply]
Lowellian (reply) 19:21, 29 September 2008 (UTC)[reply]
As mentioned, the cover holds the paper flat. Also, the cover helps with color/contrast by providing a nice white background. Without the cover closed, everything around the paper is black. Since the software is designed to look for white around the edges, finding black will cause differences in color/contrast. Newer scanners avoid this problem by having a white strip inside the machine that you don't see. I, also, have found it annoying to use the cover before. When I shut it, papers/photos move around. So, I use a small book to hold things down on the scanner. -- kainaw 11:13, 29 September 2008 (UTC)[reply]
A lot of people are paranoid about that light. Against all logic many people think it's some sort of dangerous radiation and insist on keeping the cover down. ("Better safe than sorry!") I imagine a scanner without a cover would not sell very well because it would make some people so nervous. Even if they knew intellectually that it was an irrational fear they'd find a reason to buy some other scanner.
Also, as everyone else has said, the light can be annoying in a dimly lit room, If you're standing there waiting for it to finish it can dazzle you and irritate your eyes, and the cover keeps things flat and stops them from blowing away if you're next to an AC vent. And finally it stops dust from accumulating on the glass while the scanner is not in use. APL (talk) 13:17, 29 September 2008 (UTC)[reply]
See Snow blindness and Over-illumination about having too much light. I had a look at the Interactive whiteboard article because they should have warnings attached about looking at the light, having them too bright and about problems with the eye no adjusting properly if the glare is peripheral giving rise to a temporary tunnel vision or even snow blindness if there is too much UV light. Dmcq (talk) 14:32, 29 September 2008 (UTC)[reply]
Copying thin paper with the lid open will make the copy/scan dark, because the light goes through the paper instead of bouncing back into the machine. I belive it will also worsen the problem of the text on the top side of the paper shining through. You might test this with a page from a newspaper. EverGreg (talk) 18:20, 29 September 2008 (UTC)[reply]

When blue planet?

Did we all think the planet earth was green until astronauts saw it from outer space? And if so, when did we realise the truth? Alan Rothwell (talk) 11:33, 29 September 2008 (UTC)[reply]

We knew that water was blueish and that the sky was blue for a long time before going into space, so it seems pretty reasonable one might have assumed "looks blue from there too". But the second part of your question is confusing: "if so" (i.e., "we all think the planet earth was green until astronauts saw it from outer space") then the answer is right there (when "astronauts saw it from outer space"). DMacks (talk) 13:13, 29 September 2008 (UTC)[reply]

Thank you, DMacks. That'll do. Regards Alan Rothwell (talk) 15:45, 29 September 2008 (UTC)[reply]

(Of course it wasn't astronauts who first saw the earth from outer space - it was people looking at photos from spy satellites. Before that, high altitude balloons and aircraft could tell that the planet would look pretty much blue from above.)
But anyway: for final proof, if you look back at colored atlasses and globes dating back hundreds of years, they always carefully painted the oceans blue - so I think it's pretty clear we expected them to look that way from a distance. Here, for example is an 1891 globe - and it's blue. SteveBaker (talk) 18:44, 29 September 2008 (UTC)[reply]
A child I know heard the rhyme "In 14 hundred and 92, Columbus sailed the ocean blue," and asked "What color was it before that?" Edison (talk) 19:36, 29 September 2008 (UTC)[reply]
The OP is quire correct: pre-1957, the general public thought the planet was green. This is clear from reading old science fiction, including thst written by the most scientifically literate writers. "The Green Hills of Earth," by Robert Hienlien, is an example. If nayone had asked one of those guys to analyze the actual color, most of them would have figured it out, but nobody asked. All of us who can remember the first color piictures from space can remember being startled by the fact that our planet is blue. -68.110.230.28 (talk) 02:02, 30 September 2008 (UTC)[reply]
That's not true! I vividly recall seeing the first color photos from orbit - and I don't recall anyone being in the slightest bit surprised. I don't think Heilein's title says much - he says the hills are green (which they pretty much are) - but he doesn't say that the entire planet is green - or indeed that his observation of green hills is from orbit. I repeat my earlier "proof" that globes dating back MANY hundreds of years have always had the oceans painted blue...why would they do that if they thought the world would look green from that perspective? SteveBaker (talk) 02:44, 30 September 2008 (UTC)[reply]
Please see this page. The first color image was apparently made in 1967. -Arch dude (talk) 04:43, 30 September 2008 (UTC)[reply]
I strongly disagree with the claim that people thought the earth was green before the space age. They certainly knew that most of the earth was ocean, and that oceans were blue. They also knew that deserts were not green. Note that Heinlein only referred to green HILLS. The blueness of pictures from space was startling only to the uninformed. Edison (talk) 05:31, 30 September 2008 (UTC)[reply]
Star Trek and plenty of other productions had shots of Earth (or Earthlike planets) seen from the Big Black; did they have green oceans? (I didn't have color television until 1981, so have no clear memory on this point.) The strikingly odd thing is that they had no clouds. —Tamfang (talk) 05:33, 30 September 2008 (UTC)[reply]

Digestive System

How does the Digestive system help the circulatory system function properly —Preceding unsigned comment added by 136.229.5.40 (talk) 12:11, 29 September 2008 (UTC)[reply]

Sounds like a homework question to me. Ask somewhere else. PS: the heart needs energy to function, so where does the energy come from? Food. :) 31306D696E6E69636B6D (talk) 13:25, 29 September 2008 (UTC)[reply]
Well, yes, but it sounds like a really bad homework question. Everyone knows that the rest of the body is just the liver's way of making other livers. - Nunh-huh 13:47, 29 September 2008 (UTC)[reply]
this is no place for bilious comments --Scray (talk) 02:59, 30 September 2008 (UTC)[reply]

DACs in CD players

I have just bought a new CD player with A delta sigma converter that uses a switched capacitor DAC. These are sometimes called bit stream or 1 bit DACs.

I found it sounds quite different (warmer, richer etc) from my previous CD player which used the TDA 1514 DAC chip and associated digital filter chip. Can anyone point me to articles discussing why the two types of DAC sound so different?--GreenSpigot (talk) 13:46, 29 September 2008 (UTC)[reply]

May I suggest a double blind test to establish that the effect is real? If it is, I would expect differences in the quality and calibration of the pre-amp to affect the the sound much more than the DAC. Can you ensure that all other variables are eliminated? --Stephan Schulz (talk) 14:46, 29 September 2008 (UTC)[reply]
I dont think I need to do a DBT to know that the difference is remarkably obvious to my ears. I was totally surprised as I was not expecting to hear any difference whatsoever. So what could the possible reasons be for the unmistakable difference?--GreenSpigot (talk) 17:03, 29 September 2008 (UTC)[reply]
You'd be surprised at how easier it is for people to hear a difference that isn't there, even whey they aren't outwardly expecting it. Nil Einne (talk) 18:12, 29 September 2008 (UTC)[reply]
Unless one or the other was malfunctioning, there should not be any obvious difference, since it is such high fidelity, with wide frequency response, low noise level, wide dynamic range, and freedom from clicks, rumble, scratches, hiss, wow and flutter which plagued stereo listeners in the analog age. Edison (talk) 19:38, 29 September 2008 (UTC)[reply]
I agree. There is a deep problem with serious audiophiles. In the days of analog equipment, they had a lot of fun debating the pro's and cons of various turntables, amps, etc - tricking out turntables with different cartridges, endlessly arguing over the fidelity of vacuum tubes versus transistors. Then we got digital equipment, and now, even a $20 CD player can produce audio significantly better than the human ear can resolve. This has resulted in a lot of people with no hobby. The unfortunate consequence is people spending hundreds of dollars on gold plated nitrogen-injected digital audio cables(!) and other ridiculous junk. This reviewer says that his $300 (!) digital audio cable "The brightness is no longer a problem and now I get the chance to hear the seperation of the instruments and the kind of bass that is tight as a sludge hammer." -- well, I'm surprised he's looking for his audio to sound like a sludge hammer - but for 100% certain, a DIGITAL signal isn't going to sound any different whatever on a $1 cable or on a $300 cable...that's the entire point of digital signals. If someone can convince themselves that a cable (of all things) could possibly make any difference whatever (let alone a $300 difference!) then I'm equally sure that our OP isn't really hearing a difference. An honest double-blind test is undoubtedly called for. SteveBaker (talk) 23:10, 29 September 2008 (UTC)[reply]

Is a 10% reduction of emissions reasonable?

In Australia, the Federal government's top climate adviser, Ross Garnaut, says that in 2020 a reduction of pollution levels by 10% of 2000 levels is a realistic target. WWF Australia chief executive, Greg Bourne, says that a 20% reduction is better.

http://news.smh.com.au/national/garnaut-wants-10-emissions-cut-by-2020-20080905-4a2d.html

I'm confused. I know little about global warming other than it is real. Does anybody have an argument or some short sharp facts to back up the claim that a larger than 10% reduction in pollution would be better for Australia?

What's a reasonable, realistic, economically responsible target to reduce pollution levels to?

Similarly to the above question, what about targets for the rest of the world? ExitRight (talk) 14:52, 29 September 2008 (UTC)[reply]

You may find Kyoto Protocol interesting. The UK has set a target of 60% reduction from 1990 levels by 2050 and seems to be on track, so large reductions are possible. Whether Australia can manage a 20% reduction would depend on what's happened in the last 8 years since we're nearly half way through the given time period. --Tango (talk) 14:59, 29 September 2008 (UTC)[reply]
In terms of what's beneficial - a 99.99% reduction would be beneficial from the point of view of avoiding global warming - but the larger the reduction you make, the more it costs the country to do it. Hence there is a balance between benefits to the atmosphere and detriments to the economy. Precisely where that balance is turns out to be almost impossible to calculate. Precisely how much (in monetary terms) is the extinction of the polar bear worth? We can't put a number on that. We can say how much it's going to cost to rebuild cities that get flooded by rising sea levels - and perhaps we can nail an estimate on the cost of crop losses...but because we don't know how much sea level rise there will be - we can't know. The cost to the economy also depends on how you make that reduction. If you buy technology that's already available, it costs some amount - if you throw a lot of cash into research and your country becomes not only a consumer of anti-global-warming technology but also a PRODUCER of it - then it's possible that tougher standards might actually IMPROVE the state of your economy. If you make CO2 reductions by cutting fuel consumption - you'll save money - if you do it by building damned great "carbon sequestration" plants and continue to burn carbon-based fuels - then it's going to cost you a small fortune. A lot depends too on what other countries do. If Australia makes a 20% reduction at huge expense to the taxpayer - but nobody else helps out - and Australian coastal cities get drowned and your fields turn to desert anyway - then that was a waste of money. But if everyone thinks that way - it's a self-fulfilling prophesy.
It's all very difficult - and that's precisely why we have to have the Kyoto accord (or something very much like it) - it's supposed to set standards that we're all going to work hard to meet so that we don't have this kind of debate anymore. However, since the US screwed it up by refusing to ratify it...we're back to arguing about it all over again. SteveBaker (talk) 18:35, 29 September 2008 (UTC)[reply]

heating element

what do we mean by buonet heating element. types of heating elements —Preceding unsigned comment added by 117.195.175.156 (talk) 15:08, 29 September 2008 (UTC)[reply]

Do you mean bayonet, such as http://www.secowarwick.com/aftermar/RN311-2.pdf? -- SGBailey (talk) 15:26, 29 September 2008 (UTC)[reply]

the details of callus formation

How *exactly* do calluses form? The more details the better. Is there some form of callus formation factor that gets released with repeated bruising or pressure that prevents dead skin from shedding and speeds cell death to form that protective layer? Seeing that it often occurs within mere hours of tedious work, it almost seems to me that there are some active factors that get secreted. Googling has been pretty frustrating, because they keep telling you it's the body's natural defence but they tell you nothing about the biology of callus formation. How far back up the phylogenetic tree does this reaction go? I'm also finding it hard to find any useful papers on callus formation, since they all seem to deal with various genetic transformation techniques, etc. John Riemann Soong (talk) 16:10, 29 September 2008 (UTC)[reply]

Our Callus article does not answer your question. If you do find the answer, please update the article. -68.110.230.28 (talk) 01:49, 30 September 2008 (UTC)[reply]

reg.heavy duty trailers

dear sir, 'iwish to have the address of an automobile manufacturer in austrlia who r making long and heavy duty trailers containing more than 100 tyres and controlled by electronic sensors and devices. please give information about it.

thanking u, s.r —Preceding unsigned comment added by Rmohan14 (talkcontribs) 16:24, 29 September 2008 (UTC) [reply]

Jebus, 100 tyres? that sounds like an awful lot. Not that your question has anything to do with science... Why don't you try contacting the Australian commercial vehicles department or something? Edit: i guess you're looking for something like this [19] (not Australian) although not even all those beasts appear to have 100 tyres Nil Einne (talk) 18:26, 29 September 2008 (UTC)[reply]
I believe the concept in Australia is called Road train and according to our article, the world record for the longest road train is held by Marleys Transport with 45 trailers. It does not say how many wheels but its got to be at least 45x4 and probably a lot more. There are lots of links at the bottom of the article which may help you. SpinningSpark 19:04, 29 September 2008 (UTC)[reply]
I assumed we were talking about those hydraulic-drive platforms that they use for things like moving entire buildings and such. They do have maybe 100 tyres and are driven using fancy computer stuff to keep them dead level and to allow all 100 wheels to steer. SteveBaker (talk) 21:16, 29 September 2008 (UTC)[reply]

High and low biotechnology

What exactly are "high" and "low" biotechnology? I've seen my professor use these words in my biotechnology class, but since I don't have class for a few days, I figured I should ask here instead of looking like a moron in class. Nishkid64 (Make articles, not wikidrama) 16:25, 29 September 2008 (UTC)[reply]

I don't know what they mean, and I think we can assume no one else in your class does, either, so far from looking like a moron, you'll look like a hero for being brave enough to ask! --Sean 19:10, 29 September 2008 (UTC)[reply]

Lipids

Are lipids considered polymers? Or are they simply a backbone with the "head"? --MrE1 (talk) 21:27, 29 September 2008 (UTC)[reply]

Lipid monomers (like palmitate) can be part of polymers, like cutin, cutan, and suberin. These water-insoluble substances are not yet as well characterized as many other more-familiar polymers. --Scray (talk) 02:56, 30 September 2008 (UTC)[reply]
Lipids are not polymers because they are not made of a repeating unit cell. And I think you mean "fatty acids" and not necessarily lipids, because lipids refers to all biological material that is soluble in non-polar solvents, and while it does include the "backbone and head" structure you find in a fatty acid, the class of lipids ALSO includes the steroids, like for example cholesterol. Look at the article lipid for some example structures of common lipids. Proteins and nucleic acids could sometimes be considered biological polymers, however not in the strictest sense, since the unit cells are not identical, but merely drawn from a finite set of possible unit cells (about 20 for proteins, and 4 for DNA for example). Polysaccharides are the closest thing to true biological polymers, since they do contain long chains of identical monomers. There are some biological molecules that are made of lipid monomer cells, but that's still not what you are asking above... --Jayron32.talk.contribs 03:00, 30 September 2008 (UTC)[reply]

Mate selection in gulls

What criteria does a female herring gull (or the male, if he's the one that picks and chooses in this species - I don't actually know which way round it is) use when selecting a desirable mate from a lineup of likely candidates? From my observations of these birds, their mating behaviour in general is very subtle - the male and the female just seem to start 'hanging out' together after no discernible preamble, occasionally calling out or head-flicking in unison. So, does anyone know what physical features and behavioural attributes are considered desirable to the opposite sex in this species? As I say, I've been watching gulls for years and I have absolutely no idea... --Kurt Shaped Box (talk) 21:47, 29 September 2008 (UTC)[reply]

dogs kicking back after defecation

Why do dogs sometimes, after defecation, step aside and kick back scratching the ground, several times? --Halcatalyst (talk) 22:23, 29 September 2008 (UTC)[reply]

I suspect they are responding to some ancient wolfish instinct to bury their poop in order to mask their presence when hunting. However, being dogs, they've kinda forgotten why and only half-heartedly bother. SteveBaker (talk) 22:53, 29 September 2008 (UTC)[reply]
I know Steve well enough to know what he means (and it isn't what he typed). We can't claim wolves know why they bury their poop (or that they even know they are doing it). It is an evolutionary trait that increases the chance of survival. As such, animals that evolved from wolves carry the same traits - but possibly in a much weaker form. This is just one of those cases where it is easier to say "Wolves do X because of Y" or (worse) "Evolution made wolves do X." Applying human reasoning to animals leads to false statements. Applying human reasoning to evolution (as though it were an intelligent being) leads to false statements. But, it is much easier to treat everything as human (ie: My keyboard is trying to piss me off with this stupid sticky left shift key!) -- kainaw 23:17, 29 September 2008 (UTC)[reply]
Anthropomorphism Mac Davis (talk) 23:41, 29 September 2008 (UTC)[reply]
I've also heard the alternative version that dogs do this to spread their feaces to mark their territory in tyhe same way that they urinate on every tree in sight. 190.244.186.234 (talk) 23:46, 29 September 2008 (UTC)[reply]
Well, yes. To phrase it in a more concise manner: It seems likely that wolves have an instinct to bury their poop. Domesticated dogs do not need that evolutionary adaptation and are therefore losing the instinct...but it's not entirely gone yet. It's possible that the original instinct is to spread the stuff rather than bury it. It's hard to know for sure. From what I've observed in my dogs, they are definitely trying to bury it - but they do indeed give up after a couple of half-hearted scrapes. SteveBaker (talk) 02:38, 30 September 2008 (UTC)[reply]
Well actually y'all have no idea what goes on in dogs' or wolves' heads - maybe they all deductively reason out the pros and cons of poop-burial, and it's intelligence that was evolved. But that's just me being a trollish animal-rights person. --Bmk (talk) 06:28, 30 September 2008 (UTC)[reply]

September 30

Photosynthetic Animal

Hey. I'm trying to remember a certain animal that I think I recall seeing on a nature documentary, it was like, some kind of newt or something. But its most remarkable feature was that it shared a symbiotic relationship with algae that lived just underneath its skin, the algae would absorb the sunlight and effectively create food for the newt to live off of. Does anyone have an idea of what animal I'm thinking of? 62.49.131.66 (talk) 00:56, 30 September 2008 (UTC)[reply]

Elysia viridis ? It's a sea slug and I don't think it assimilates the intact algae, only chloroplasts. I've never heard of a "solar-powered newt" so to speak, but I'd sure love to know if there exists one :) --Dr Dima (talk) 01:48, 30 September 2008 (UTC)[reply]

Power consumption vs. hour of day

I want to know, in a typical home, what hours of the day tend to show peak electrical power usage, what hours of the day show minimal electrical power usage, and the relative differences throughout the day. Basically, I'd like to see a graph of power consumption vs. hour of day in a typical home in a city. —Lowellian (reply) 02:44, 30 September 2008 (UTC)[reply]

It is difficult to have a typical home by hour of day. Most power consumption occurs when people are at home. So, you have to start by generalizing to the point of being incorrect by deciding which hours of the day people are at home (and awake) in a "typical" home. -- kainaw 04:43, 30 September 2008 (UTC)[reply]
Here's some articles I found through a google search: [20]. About 1/4 of the way down the page is a graph of the power consumption over a single winter day in Humbolt County, California. This one:[21] shows the last 24 hours power consumption in Ontario, Canada. This one: [22] (see figure 2) shows an average graph by season, though I don't see that one's methodology. There's 3 for ya. Good luck! --Jayron32.talk.contribs 04:55, 30 September 2008 (UTC)[reply]
The power consumption through the day will vary by season and by region. See Load profile. A Google Book search shows lots of books which discuss such load profiles or load curves. Your own utility doubtless has data on residential load curves which they might provide you. Edison (talk) 05:28, 30 September 2008 (UTC)[reply]
(ec)I wrote a paper on the energy saving benefits of daylight savings time (there isn't one, by the way), and the timing of people's power usage was huge part of it. Suffice to say that it varies with climate, season, and culture. Some people need to heat their houses all night and some need to air condition it all day. I don't know what you are using it for, but you might be interested to know that industrial power consumption is more than 3 times larger than domestic consumption. Plasticup T/C 05:29, 30 September 2008 (UTC)[reply]
Off-topic, but since you mention daylight-saving time - now that the powers-that-be have decided (solely on energy-saving grounds mind you) to extend DST to cover more than half the year, shouldn't we just call it "Time"? The thing we do in winter should now be called "Daylight Deficit Time". Franamax (talk) 06:19, 30 September 2008 (UTC)[reply]

Tyvek house wrap

I live in central Ontario, Canada. My house is about 30 years old. It is a two story with brick on the first level and siding on the second. My question is this: is it worth the effort and expense to remove the siding, wrap and seal the second level with Tyvek and then put the siding back on? Modern houses are wraped in Tyvek but I don't believe that they were 30 years ago. Thoughts? —Preceding unsigned comment added by 209.161.212.209 (talk) 04:27, 30 September 2008 (UTC)[reply]

If it were my house, I would not do it. I question what the payback period would be for such a retrofit. Edison (talk) 05:23, 30 September 2008 (UTC)[reply]
I wonder if there are tax deductions for improving the insulation in your house. That might be something to consider. Plasticup T/C 05:30, 30 September 2008 (UTC)[reply]
Tyvek (TM) forms an air-tight/water-proof barrier, so the question is whether your existing siding is inadequate to form the air-seal. Call Ontario Hydro (Hydro One now, or your local electric utility) and ask if they still have the cut-rate efficiency assessment programs. Whichever heating supplier you have (such as Enbridge) is a good starting point too. If you're going to pull off the siding, you should also be thinking about putting on 40mm or 50mm of that blue styrofoam insulation. (You'll need 2" corner-caps if you do that though) Franamax (talk) 05:42, 30 September 2008 (UTC)[reply]
And rather than strike the above, I'll note instead that if it's an Ontario house, 30 years old, it will already have an air/moisture-seal - that black wax-coated paper you used to always see on houses as they were being built, before Tyvek figured out how to print their brand name in big letters across the whole sheet. If you have no moisture seal at all, you may have a problem - pry up a bit of the siding and look for the black paper. Franamax (talk) 05:51, 30 September 2008 (UTC)[reply]